Главная Настройка Mobile Контакты NSFW Каталог Пожертвования Купить пасскод Pics Adult Pics API Архив Реквест доски Каталог стикеров Реклама
Доски


[Ответить в тред] Ответить в тред

Check this out!


<<
[Назад][Обновить тред][Вниз][Каталог] [ Автообновление ] 502 | 53 | 202

Тупых вопросов тред Аноним 19/01/18 Птн 17:16:39  429247  
А то прошлый тонет уже.

Начну. Как можно использовать многомировую интерпретацию квантовой механики чтобы получить сверхспособности?
Аноним 19/01/18 Птн 17:17:02  429248
>>429247 (OP)
Простите за проёбанную запятую.
Аноним 19/01/18 Птн 17:21:29  429250
Предыдущий тред (ещё жив): https://2ch.hk/sci/res/427340.html
Белый чай ku 20/01/18 Суб 13:36:30  429297
У меня в голове сидит ооооочень давно теория. Смысл в том что может быть телепортация чего либо напрямую зависит от информации места назначения, допустим если человек летит в страну N в которой он ни разу не был, то он получает много детально точной информации о стране N ( например в аэропорту он подсчитал кол-во бутылок в автомате ) и именно это определяет то что он в этой стране, знаю все очень сыро звучит, но вот сам вопрос, как думаете возможно ли существование такого мышления ?
Аноним 20/01/18 Суб 14:11:17  429302
>>429297
Ваще нечитабельно, блядь, но как я понял - ты предлагаешь тупо накачать башку инфой
по узнаваемым ключевым элементам, причём так
чтобы создавалась иллюзия того, что чел реально был в какой-то стране,
вместо того чтобы волочить его туда в реале.
И сидишь потом такой короче, ротируешь в памяти инфу себе,
и удивляешься от того, что ты "РЕАЛЬНО БЫЛ" там, там, вооот там и ДАЖЕ ТАМ!
Но раз речь идёт именно о телепортации, и раз она подразумевает именно перемещенение в пространстве,
то я так полагаю ты думаешь что сам перенос инфы туда может ею и являться.
Но для телепортации надо же носитель какой-нибудь (или связь с ним),
а иначе телепортации никакой не будет, даже если ты сидя там -
будешь ощущать иллюзию телепортации куда-то туда в эту твою страну N.
Аноним 20/01/18 Суб 17:52:17  429315
Реквестирую годные не научпопные книги учебники? по эволюции растений, их жизненных циклам и проч.
Аноним 20/01/18 Суб 18:47:31  429318
>>429302
>>429297
Зачем находиться в какой-то точке пространства?
Чтобы получать информацию и влиять на пространство и объекты.

Человеческое мышление просто обрабатывает информацию, не важно, откуда она идёт.

https://www.youtube.com/watch?v=4c1lqFXHvqI
Аноним 20/01/18 Суб 19:53:47  429321
>>429318
Говоришь так, как будто для того чтобы получать информацию и влиять на пространство и объекты -
надо действительно находиться там,
а не находиться где-то в другом месте с иллюзией присутствия там,
и имея при этом связь с функциональными элементами, которые могли бы делать это - ТАМ.
К тому же, если не нужно влиять ни на пространство, ни на объекты - вообще ни на что,
а просто нужно получить какую-то информацию оттуда - то находиться там тем более не обязательно,
а достаточно смоделировать эту информацию ОТТУДА и представить её сознанию, не находящемуся там.
Аноним 21/01/18 Вск 00:27:14  429330
Являются ли сновидения суперпозицией мозга по отношению к бодрствованию и отключке?
Аноним 21/01/18 Вск 12:38:29  429352
Социо-математический вопрос.
Предположим Что на земле остались только бабы и Нное количество мужчин.
Каково минимальное количество мужчин, при идеальных условиях чтобы следующее поколение было не меньше предыдущего.
Аноним 21/01/18 Вск 13:01:44  429354
>>429247 (OP)
Ну короче смотри. С помощью высоких технологий будущего перемещаешься в параллельную реальность, и распадаешься на элементарные частицы из за невозможности существовать там макро элементов.
А если серьёзно то просто приобрети ружьё и станешь самым сильным в лесу.
Другими словами интерпретация это не синоним "спупер девайс" Хватит мангу читать.
Аноним 21/01/18 Вск 21:31:56  429396
Пытаюсь вырастить кедры в домашних условиях, месяц назад замочил на неделю орешки потом высадил в смесь песка и грунта, в холодильник на зиму решил не ставить, пока ничего не взошло, вскрыл несколько небольших внутри орешка просматривается столбик, через сколько могут начать всходить и как можно ещё по-содействовать если у кого есть опыт?
Аноним 24/01/18 Срд 12:02:00  429551
>>429352
Если молодой и горячий кауцказец - хватит и одного.
Аноним 24/01/18 Срд 15:56:19  429565
>>429318
Так и смотри видео на ютубчике про Тибет - будешь в тибете по своей логике, но это не называется телепортацией, ты просто подменяешь понятия.
Аноним 24/01/18 Срд 15:57:01  429566
>>429551
Дуачую
Аноним 24/01/18 Срд 16:05:23  429569
>>429352
Здесь множество факторов играют важную роль вплоть от развития науки до культуры. Если бабы смогут в искусственное оплодотворение, то очень много мужчин им не потребуется. Допустим у нас идеальные условия в которых живут люди. Великолепная медицина, экономика, все женщин беременеют сразу и согласны на экстракорпоральное оплодотворение. Давай считать. В среднем у мужчины выделяется 2-5 мл эякулята. При получении спермограммы приемлемыми показателями эякулята являются минимум 2 мл эякулята, в которых концентрация сперматозоидов 20 млн/мл. От этих данных и будем отталкиваться. Существует 4 категории подвижности сперматозоидов, нормальным считается 50% сперматозоидов категории А и Б от общего числа. Из приведенных данных получаем за один акт эякуляции 2 мл спермы, в которой 40 млн сперматозоидов из которых для оплодотворения пригодны только 20 млн сперматозоидов. Мужчинам перед сдачей спермы для анализа рекомендовано воздержание от половой жизни и мастурбации в течение 2-7 дней. Возьмём среднее в пять дней. Допустим, что нашему мужчине потребуется 5 дней для воспроизводства 2 мл эякулята в которых 20 млн сперматозоидов, пригодных для искусственного оплодотворения. 365/5 = 73 дня. 73 * 20 000 000 = 1 460 000 000. 1 460 000 000 пригодных для оплодотворения сперматозоидов в год. На Земле живут 7 млрд людей из который 3,5 млрд женщины. Т.е. потребуется 3 года, чтобы один мужчина смог оплодотворить каждую женщину и ещё с лихвой останется на повторное оплодотворение.
Аноним 24/01/18 Срд 16:09:22  429571
По химии вопрос. Почему при реакции например натрия с водой в щелочи NaOH остается водород? То есть почему один атом водорода заменяется натрием, а второй нет?
Аноним 24/01/18 Срд 16:20:43  429575
>>429571
Ты рассматриваешь идеальный пример. В жизни все в принципе не так как в формулах.
Но в примере банально за счет электроотрицательности.
Na(1+) + O(2-) + H(1+)
Два атома берется з молекулы воды, один остается, натрий более активный и связь с ним сильнее, поэтому водород идет нахуй как никому не нужный.
Аноним 24/01/18 Срд 16:21:00  429576
>>429571
Ты рассматриваешь идеальный пример. В жизни все в принципе не так как в формулах.
Но в примере банально за счет электроотрицательности.
Na(1+) + O(2-) + H(1+)
Два атома берется з молекулы воды, один остается, натрий более активный и связь с ним сильнее, поэтому водород идет нахуй как никому не нужный.
Аноним 24/01/18 Срд 16:35:30  429579
>>429575
>Но в примере банально за счет электроотрицательности.
>Na(1+) + O(2-) + H(1+)
Почему вот так не получается?
Na(1+) + O(2-) + Na(1+)
Аноним 24/01/18 Срд 16:44:18  429580
>>429579
С этого стоило начать. Потому что H(1+) и OH(1-) - Это разные ионы, первое сосет хуи, второе очень активное и любит детишек.
Именно из-за второго щелочь такая едкая.
Аноним 24/01/18 Срд 16:55:50  429581
>>429580
То есть в NaOH водород по другому связан с кислородом, чем в Н20?
Аноним 24/01/18 Срд 17:20:35  429582
>>429581
>>429581
Да, в разных молекулах одни и те же связи могут быть разными за счет разной степени смещения электронных облаков. Но тут уже приходится усложнять, а я вроде как стараюсь без этого.
Если хочется разобраться детально - глянь что такое гибридизация.
Аноним 24/01/18 Срд 17:37:00  429584
>>429354
>Хватит мангу читать.
Есть предположение что любая сказка может стать реальностью при стремящемся к бесконечности прогрессе. Например, для создания Дев Розена осталось реализовать квантовые нейросети внутри кристалла.
Любая развитая технология неотличима от магии => любая магия может быть реализована с помощью высокоразвитой технологии.
Аноним 26/01/18 Птн 12:23:35  429667
>>429247 (OP)
>>429248
Сморозил тупость, извиняется за запятую. Но как не поддержать собрата по разуму: можно почти мгновенно перемещаться в любую точку 3х мерного пространства, через 4ое, фармить халявную энергию.
Аноним 26/01/18 Птн 12:47:30  429668
>>429571
Потому что оторвать второй водород от OH- сложнее намного. А у тебя выбор у натрия. либо у воды брать. Либо у OH-. Воды 1) больше. 2) Она нейтральная.
Если ты не оставишь натрию выбора. И начнешь его греть с NaOH. То получится оксид и водород.
Аноним 26/01/18 Птн 13:31:58  429670
Наверное охуенно тупой вопрос: если объект А движется со скоростью в 60% от скорости света, а ему навстречу летит объект Б с такой же скоростью, получается, они движутся друг к другу быстрее скорости света? Если скорость относительна, почему она не может быть выше скорости света?
Аноним 26/01/18 Птн 13:35:15  429672
4397.jpg (1498Кб, 1920x1280)
Допустим, вокруг Солнца построили сферу Дайсона, радиусом больше, чем расстояние от Солнца до Земли. Какого цвета будет небо? Днём, ночью? Если диаметр сферы будет как у орбиты Марса, Юпитера, Плутона?
Аноним 26/01/18 Птн 13:39:16  429673
image.png (3Кб, 218x91)
>>429670
Закон сложения скоростей. выходит 0,823с

Аноним 26/01/18 Птн 13:39:54  429674
>>429672
загугли, какое количество люмусов излучает слце, поищи потом степеь отражения поверхности от железа какого нить
будет светло, мягко говоря
Аноним 26/01/18 Птн 15:44:37  429680
https://www.youtube.com/watch?v=JhHMJCUmq28
Кто-нибудь может объяснить как работает квантовый компьютер? В научно-популярном говне твердят одно и тоже, последовательность кубитов может хранить гораздо больше значений, чем последовательность бит. Но как вообще кубит может что-то хранить, если он выдает рандом нахуй?

>>429664
Логика тоже своя? Можно в принципе объяснить, как кубит, выдающий рандомное значение, позволит что-то хранить и производить вычисления?
Представьте, что у вас электрическая пила и при нажатии на копку, она либо выключается либо нет. То есть в критический момент можно вплоть до ранения дойти, разве есть в технике что-либо работающее на принципе неопределенности?
Аноним 26/01/18 Птн 16:01:27  429682
13674313252880.jpeg (53Кб, 699x629)
>>429247 (OP)
Щас буду нести дикую хуйню, только не бейте лучше обоссыте
Вообще в душе не ебу как устроены электросети и как коммунальщики определяют с какой квартиры идет тот или иной расход электричества. Я себе представлял раньше так: для упрощения пусть дом состоит из двух квартир. Васян из 1 кв. потребляет 100 кВт⋅ч в месяц, а Стасян из 2 кв. 200 кВт⋅ч. Тетя Срака в ЖЭКе видит, что весь дом за месяц расходует 300 кВт⋅ч. Так как она определяет сколько потребил Стасян, а сколько Васян? Или показания счетчиков с каждой квартиры "идут" по отдельности, а не суммируются? Как это технически реализовано? Отдельный провод для Стасяна, отдельный для Васяна?
Аноним 26/01/18 Птн 16:05:07  429683
>>429682
>Тетя Срака в ЖЭКе видит, что весь дом за месяц расходует 300 кВт⋅ч.
Общедомовой счетчик.
Аноним 27/01/18 Суб 11:27:46  429735
>>429247 (OP)
Погружаешься в 4Д мультиверс-пространство, в котором скорость света в 100 раз выше чем в том 4Д мультиверс-пространстве откуда ты прибыл.
Пилишь там на 0.05с, разгоняяв астеройд из обогащённого дейтерия, который бохат гелием-3.
Всплываешь в исходном 4Д пространстве. Запиливаешь колонию на подходящей планете около годной звезды.
Шлёшь земляшкам сети-сигналы на русском:
"У кого нет варпа - могут идти в жопу!".
Проигрываешь с одномирового скама, попивая сок у себя на планете, в варп-телескоп виден антисетишизик, развивший вторую космическую скорость за счёт пригоревшего афедрона.
Аноним 27/01/18 Суб 14:10:52  429746
>>429565
По моей логике я могу подключиться к квадрокоптеру, летающему по Тибету, и это уже будет похоже на телепортацию.
Я буду получать информацию, которую хочу, с какого хочу ракурса, а приделав манипуляторы, ещё и влиять на неё.
И по моей логике, если сделать что-то типо виртуальной машины, которая будет ставить в соответствие информацию с сенсоров с квадрокоптера к моим сенсорам высоты, запаха, тактильных ощущений и всего прочего, а не только зрения, то это будет равно телепортации.

https://youtu.be/37GtkTZBWNw?t=406
Аноним 27/01/18 Суб 16:18:31  429749
Теси
Аноним 27/01/18 Суб 19:29:58  429753
zKF5Wl8Bg-Q.jpg (40Кб, 604x334)
О корпускулярно-волновом дуализме.
Как не понимал на младших курсах, так и сейчас не понимаю.
Лабораторный практикум.
Проведем опыт по доказательству хуйня-нейм. Так-с, так-с, что тут у нас? Свет? Попробуем объяснить опыт с точки зрения волновой теории. Нет, что-то не сходится. А, ну давайте тогда попробуем квантовую. О, вот теперь заебца!
В других случаях наоборот. И нихуя не понятно почему мы выбираем тот или иной подход.
Существует ли какое-то логическое объяснение помимо совпадения/несовпадения с результатами опытов, обобщенная теория?
Может литература есть, где это объясняется популярно, не требуя особых знаний математики? Я у мамы инженер-недоучка. Буду признателен.
Аноним 27/01/18 Суб 19:42:15  429754
>>429753
>Существует ли какое-то логическое объяснение помимо совпадения/несовпадения с результатами опытов, обобщенная теория?
В этом суть физики: эксперимент > логика. Даже сама логика есть лишь инструмент по описанию природы, и если что-то не сходится значит логика ниправильная, но не природа.
Аноним 27/01/18 Суб 19:53:56  429755
>>429754
Попробую переформулировать вопрос.
Если ли источники, в которых просто (без особых математических сложностей) поясняется как выбирать описательный подход ДО, а не ПОСЛЕ проведения опыта, предсказать результат, считая, что заранее мы не знаем, как свет поведет себя в конкретной ситуации. Или же более корректная теория, а не та, которой нас учат?
Аноним 27/01/18 Суб 20:00:27  429758
>>429746
Смотря, что ты вкладываешь под понятие телепортации. Телепортация созания - ну, может быть, но это никак не будет телепортацией твоего настоящего, физического тела.
Аноним 27/01/18 Суб 21:09:08  429760
>>429758
Я вот это имею в виду.
>Зачем находиться в какой-то точке пространства?
Чтобы получать информацию и влиять на пространство и объекты.

Понятное дело, что если ты хочешь телепортироваться, чтобы избежать пиздеца в текущей точке пространства для своего обработчика информации - мозга, то такое не подходит.
Аноним 27/01/18 Суб 22:32:45  429763
>>429755
Ты же наверное в курсе, что сейчас концепция корпускулярно-волнового дуализма устарела. Дуализма уже не хватает. На смену пришло обобщения следующего уровня - формулировка квантовой теории через интегралы по траекториям.
Аноним 27/01/18 Суб 22:38:30  429764
>>429760
Ну это такое все равно, ты не сможешь попить водицы из горы Фуджи, отправив дрона или еще что-то, так что это и влиянием на что-либо не назвать
Аноним 27/01/18 Суб 23:12:03  429766
>>429764
Ты понимаешь, что суть влияния на среду - получение дополнительной информации?

Если ты хочешь попить водицы, чтобы провести её химический анализ - сделай такой сенсор для квадрокоптера.

Гипотетическиймозг вполне может лежать в банке в лаборотории, а сенсоры ездить по Марсу.
Аноним 27/01/18 Суб 23:15:36  429767
>>429766
Ну, я думаю наука в конечном счете к этому придет, ибо мир большой, а человечек маленький.
Но вероятность того, что люди себя уничтожат больше, чем вероятность построить такое будущее.
Аноним 28/01/18 Вск 17:09:53  429804
В общем, перекатываюсь из /un к вам, потому что там сидит сборище поехавших чсвшных пидоров. В общем, хочу подготовиться к ЕГЭ по физике, но я такой еблан, не знаю абсолютно ничего. Пробовал сначала вкатиться, читая тренажёр Лукашевой и Чистяковой, но обосрался, потому что там есть только решение задач, в которое вкатиться без теории невозможно. Начал читать элементарную физику Ландсберга в трёх томах, потому что нужны основы физики. Прочитал статику, вроде полёт нормальный. Продолжать его читать? Или вешаться нахуй?
Аноним 28/01/18 Вск 18:24:06  429805
>>429247 (OP)
Что привлекает людей в математике? Абстрактность, оторванность от реальности? Объясните, пожалуйста.
Манька 28/01/18 Вск 18:35:23  429806
>>429247 (OP)
Что привлекает людей в russian language? Разговорчивость, описание реальности? Просветлите, пожалуйста.
Аноним 28/01/18 Вск 18:41:47  429807
>>429806
Я не троллил, а задал серьезный вопрос.
Манька 28/01/18 Вск 18:45:09  429808
>>429807
Sorry, Iam not umberstanding, russian is not my родной language.
Аноним 28/01/18 Вск 18:48:32  429809
>>429808
Ясно, еды ты не получишь
Аноним 28/01/18 Вск 19:00:35  429810
>>429806
Незнание других языков? Или каких людей ты имел в виду?
Аноним 28/01/18 Вск 19:31:50  429811
>>429809
>>429810
So many questions you don't try to think.
Аноним 29/01/18 Пнд 01:33:07  429818
Почему машинку не могли изобрести веке в 1? Там же все элементарно.
Аноним 29/01/18 Пнд 01:33:27  429819
>>429818
печатную машинку
Аноним 29/01/18 Пнд 10:43:08  429829
>>429818
Я вот не могу изобрести, и для меня она такой же сложный и непонятный механизм, как и механические часы.
Аноним 29/01/18 Пнд 13:46:29  429843
Получится ли рассмотреть обстановку в комнате в соседнем здании днём, при отсутствии штор/тюля и отключенном свете в наблюдаемой комнате, если смотреть туда через длинную "зачернённую" изнутри трубу, которая отсечет для глаза обзор на более светлые (чем внутренность комнаты) участки стены дома, что (должно бы) вызвать увеличение чувствительности глаза?
Аноним 29/01/18 Пнд 13:53:11  429844
>>429804
>Начал читать элементарную физику Ландсберга в трёх томах, потому что нужны основы физики. Прочитал статику, вроде полёт нормальный.
Читай обычные (то есть нет, не обычные, а для "классов с углубленным изучением...") школьные учебники, в вузовских 95% информации, которая для сдачи егэ не пригодится и вообще в программу не входит. И задачи решай, одного знания законов тебе на егэ только для первой части хватит.
Аноним 29/01/18 Пнд 14:01:04  429845
>>429396
Тебе в /sad/
Аноним 29/01/18 Пнд 14:09:40  429846
>>429804
Продолжай читать хорошие книжки и забей на егэ. Сдашь, если очень хорошо подготовишься.
А если учить с упором на егэ - пополнишь ряды таких же чсвшных пидаров, которые не будут знать, что делать после универа.
Аноним 29/01/18 Пнд 14:43:50  429849
>>429843
Окно выхватывает слишком маленький квадрат обзора комнаты чтобы говорить об обстановке. А так все надо учитывать - расстояния, линзы, длинна трубы, дальность дома, солнечно или облачно, насколько зрачок расширен, и что есть зачернённость..
Аноним 29/01/18 Пнд 15:11:22  429850
15122402702420.jpg (141Кб, 704x1000)
>>429247 (OP)
Собираюсь конкретно угарнуть с Психогенетики. Прям по хардкору. Какю посоветуйте литературу чтобы вкатится, кого слушать, на какие лекции ходить и так далее. С меня как всегда.
Аноним 29/01/18 Пнд 18:17:31  429859
uchimsja-podnim[...].png (330Кб, 600x400)
>>429247 (OP)
Анон, как научиться шевелить бровями по отдельности, чтобы получилось?
Аноним 29/01/18 Пнд 21:13:01  429865
>>429247 (OP)
если пирамиды построили инопланетяне то почему из естественого камня, а не из синтетических хороших материалов?
Аноним 30/01/18 Втр 00:17:21  429873
>>429865
Если у тебя чешется очко, почему ты чешешь его руками, а не заказываешь реалистичную чёрную дилду с али?
Аноним 30/01/18 Втр 14:00:58  429893
14984183240881.jpg (359Кб, 640x480)
Объяснение почему при прохождении тока не разрушаются связи проводника, ведь из химии известно что электроны обуславливают валентность и почему при добавлении ещё электронов связи не должны перестраиваться мне не совсем понятно, мог бы это кто-нибудь понятным языком растолковать как для школьника?
Аноним 30/01/18 Втр 16:53:27  429899
"объясните природу мыльного пузыря"
У меня задание по физике, помогите.
Мразь физичка ставит тройку за списывание с википедии. Как описать красоту мыльного пузыря?
Аноним 30/01/18 Втр 17:12:03  429900
>>429850
бамп реквесту
Аноним 30/01/18 Втр 21:27:28  429904
Поясните за предел Канторовича
Аноним 30/01/18 Втр 22:13:58  429907
>>429846
Так я хочу пока на егэ упор, чтобы поступить, а там уже читать другое по физике, типа Фейнмановских лекций и Сивухина. Мне кажется, так будет наиболее рационально, не?
Аноним 30/01/18 Втр 22:16:50  429908
>>429844
>Читай обычные (то есть нет, не обычные, а для "классов с углубленным изучением...") школьные учебники
Можешь посоветовать авторов?
Аноним 30/01/18 Втр 22:32:12  429909
>>429908
Мне в свой время норм зашли Бутиков и профильный Мякишев (это в качестве учебника). В качестве задачника - Козел норм, если хочется по хардкору - можно взять Савченко, но это уровень школьных всеросов.
Аноним 31/01/18 Срд 00:07:49  429910
Нигде не нашёл инфы, может, кто в курсе. Сколько в % от общего времени использования электромобиля он может выдавать максимальную мощность? 20, 30, 40, 50, 80?
Я как понимаю, всё упирается в то, насколько быстро можно остудить двигатель, батарею и прочую малафью до нормальной температуры. А-то вот пишут пиковую мощность, а хуй знает, сколько можно её юзать-то в итоге. Вот у двс тачек как бы хоть постоянно газ в пол хуячь.
Ведь зная % использования пиковой мощности, можно было бы рассчитать эквивалентную мощность электромобиля, чтобы можно было сравнить с обычным автомобилем.
Максимум нахожу инфу, что может выдаваться пиковая мощность от 30 до 60 секунд. А что потом, а через сколько он опять сможет выдавать пик - секрет, что ли?
Аноним 31/01/18 Срд 00:34:04  429912
>>429910
100%. Охлаждение принудительное видел в электромобилях? Верно, его нет.

Тут всё упирается в эксплуатационные хар-ки используемых литиевых батарей.
Аноним 31/01/18 Срд 01:30:14  429913
>>429912
>Охлаждение принудительное видел в электромобилях?
Как бы да. Жидкостное, с радиаторами, которые охлаждает воздух, нагнетаемый вентилятором. Ну, как и у ДВС. И батареи даже охлаждаются.
Бля. Но меня сейчас осенило. Когда нагуглил часть дискуссии на английском.
ДВС ведь тоже если юзать газ в пол - ему будет не очень. И его тоже не часто газ в пол юзают. То есть в нормальном режиме пользования транспортным средством в принципе и похуй, какой там двигатель, электрический или двс. Собственно, там так и написали и я согласился.
Аноним 31/01/18 Срд 01:44:54  429914
>>429913
>Как бы да. Жидкостное, с радиаторами, которые охлаждает воздух, нагнетаемый вентилятором. Ну, как и у ДВС.
Как бы манда. У теслы, например, пассивный охлад, хотя тоже с хладагентом.
>И батареи даже охлаждаются.
А кроме батарей ничего и не нагревается в машине. Батареи слабое место.
Аноним 31/01/18 Срд 02:05:57  429915
>>429914
>пассивный охлад
Пассивный охлад - это радиатор без вентилятора, без помпы, если используется жидкость.
>А кроме батарей ничего и не нагревается в машине.
Двигатель сгорит без охлаждения через секунды, инвертер тоже.
Аноним 31/01/18 Срд 02:10:38  429916
aV3O9e2700b.jpg (59Кб, 700x487)
Есть такая штука как тяжелая вода - D2O. На вид и вкус она такая же как обычная? Может ли человек выпив тяжелой воды понять что он выпил что-то не то, не обычную воду?
Аноним 31/01/18 Срд 03:12:54  429917
>>429915
>Двигатель сгорит без охлаждения через секунды, инвертер тоже.
Это твои спекуляции, ни на чём не основанные. Что значит "без охлаждения"? Они блядь обдуваются ветром со скоростью 40 миль в час.
>Пассивный охлад - это радиатор без помпы
Это попытка залезть в терминологическую залупу. Мы ж тут не за этим.
Аноним 31/01/18 Срд 08:49:12  429925
>>429916
Дейтерий - стабильный изотоп водорода, радиации от тяжелой воды не будет, также как и от полутяжелой (HDO),
но на химические реакции влиять может: https://ru.wikipedia.org/wiki/Тяжёлая_вода#Биологическая_роль_и_физиологическое_воздействие
Молярная масса у неё другая... 20,0276 г/моль у D2O и 19,02140 у HDO.
Алсо, D2O может плавно переходить в HDO и H2O - выводясь из организма: https://ru.wikipedia.org/wiki/Изотопный_обмен
Аноним 31/01/18 Срд 11:05:05  429929
>>429910
>Вот у двс тачек как бы хоть постоянно газ в пол хуячь
Проснись маня, ты серишь, на постоянно газ в пол на 90% паспортной мощности - были расчитаны только авиационные поршневые двс.
В их случае охлаждающая способность есть продукт площади радиаторов и объёма обтекающего их воздуха, последний же есть третья степень, куб, ебёныть, линейной скорости аэроплана в диапазоне от 200 до 750 км/ч.
https://www.youtube.com/watch?v=H7N-6YmpbuQ
Аноним 31/01/18 Срд 11:10:47  429930
>>429893
> не разрушаются связи проводника
Какого проводника? Какие еще связи должны перестраиваться, например, в металле?
Про химию прочитай про электролитическую диссоциацию и проводники второго рода.
Аноним 31/01/18 Срд 11:11:24  429931
>>429899
>Как описать красоту мыльного пузыря?
Спроси у училки по литре
Аноним 31/01/18 Срд 11:18:58  429932
>>429916
Нет, все также.
Аноним 31/01/18 Срд 11:19:04  429933
>>429929
Я уже понял, что слишком круто.
Тогда сколько времени обычно используют внешнюю скоростную характеристику от общего числа времени/пробега? 20% где-то? По сути она нужна для интенсивных обгонов/разгонов.
>>429917
>Что значит "без охлаждения"?
Значит, что он тупо никак не охлаждается, ни вентилятора, который внутрняку обдувает, ни жидкости, которая циркулирует и тепло уносит. Более-менее продвинутые используют жидкость как переносчик тепла. Это уже довольно эффективно. Ну им нужен охлад какой-то, как и всему. У теслы там всё вообще самое топовое. У лифа, например, есть радиатор с вентиляторами, но также жидкостное охлаждение. Но у теслы помпа точно есть.


Аноним 31/01/18 Срд 11:24:36  429934
Screenshot.jpg (46Кб, 903x572)
>>429910
>Вот у двс тачек как бы хоть постоянно газ в пол хуячь.
Ты в чём-то подобен пикрилейтеду, очент похоже дрочило растопырил.
Аноним 31/01/18 Срд 11:42:37  429936
>>429934
Иными словами, на электромобили больно сильно наезжают, забывая, что двс тоже не подарок с теми же проблемами.
Аноним 31/01/18 Срд 11:45:35  429937
>>429933
>Тогда сколько времени обычно используют внешнюю скоростную характеристику от общего числа времени/пробега? 20% где-то? По сути она нужна для интенсивных обгонов/разгонов.
В нормально устроенной цивилизации, не состоящей из насмотревшихся топгира великовозрастных имбецилов и эффективных нефтепереработчиков - это вторичный параметр. Эта цивилизация устроена немношк по другому, там почти нет интенсивных разгонов - потому что 95% её живёт на равнинах, а не на горах-холмах, и почти нет обгонов - потому что у всех одинаково надроченый ИИ-автопилот, и ему поебать на хотелки Ивана Говнова.
Основной параметр её транспортной системы - максимальный пробег при минимальном расходе на него энергии, т.е. при поездке из пункта А в пункт Б - график скорость vs время будет близок к трапеции без большего основания, а не к прединфарктной кардиограмме.
Сюрпрайз, но крыша трапеции скорее всего там окажется довольно низко, между 80 и 100 км/ч, как не изъёбывайся с аэродинамикой повозок в духе почившей Aptera.
Аноним 31/01/18 Срд 12:59:03  429939
Существует русскоязычный аналог американского https://arxiv.org?
Аноним 31/01/18 Срд 15:29:16  429946
>>429909
Спасибо! Но чего-то у Козела и Савченко много задачников. Я даже на один олимпиадный спотыктнулся. Да и у Бутикова курс физики разбит на 3 книги, у Мякишева на 5 книг... Как же я долго буду только их читать, эхэх.
Еblans 31/01/18 Срд 16:33:11  429950
20180131163356.jpg (1303Кб, 2160x2160)
Бля, математики, выручайте нахуй ааа!!
Второй или третий номер.позя ^_^
Аноним 31/01/18 Срд 19:06:44  429953
1243145667.jpg (56Кб, 800x533)
>>429930
Ничего не понимаю, нигде про это ничего не сказано, ни кто не знает, чтож такое то.
Как же этот тупой ток в проводниках первого рода проходит, там ведь все уровни электронов по идее же заняты, и по химии говорили что электроны не могут находится вне определённого энергетического уровня, их там просто быть не может - они сразу возникают на конкретном уровне, как же они проходят тогда? Если они идут по следующему уровню это как-то меняет структуру краткосрочно или они выталкивают электроны со своих мест сами на них становятся и так далее? Или как объяснить почему они не разрушают материал.
Аноним 31/01/18 Срд 19:42:03  429956
>>429953
Ты пытаешься в кванты, не зная квантов. Серьезно. Пока я не стал читать нормальные книги. Я никогда бы непонял. ЛЮБОЕ объяснение. Это как объяснять плоскоземельщику часовые пояса.
Воспользуйся классической аналогией.
Аноним 31/01/18 Срд 20:03:29  429957
>>429956
Но ведь Земля плоская, это все знают.
Попробуй еще раз.
Аноним 31/01/18 Срд 21:16:35  429961
>>429907
Рационально - не поступать в универ долбоёбом. Хотя тебе решать, это всего лишь моё мнение.
Аноним 01/02/18 Чтв 05:08:15  429970
>>429247 (OP)
Обязательно ли разгонять частицы в коллайдере, или можно просто с антиматерией аннигилировать и получать такие же результаты?
Аноним 01/02/18 Чтв 10:00:15  429976
>>429933
>У теслы там всё вообще самое топовое.
Жаль только, что половина машин бракованными сходят с конвейера. Не в коня корм?
Аноним 01/02/18 Чтв 10:29:28  429977
>>429970
Это разные процессы как бы.
Аноним 01/02/18 Чтв 10:39:55  429978
>>429953
>Как же этот тупой ток в проводниках первого рода проходит, там ведь все уровни электронов по идее же заняты
Так в этом и суть, электроны там свободные, имеют свои права и голосуют и могут находиться не только на орбитах.
> электроны не могут находится вне определённого энергетического уровня, их там просто быть не может
Могут, и переходят. Светодиод светит как раз из-за перехода электронов с одного уровня на другой.
> Если они идут по следующему уровню это как-то меняет структуру краткосрочно или они выталкивают электроны со своих мест сами на них становятся и так далее?
Поздравляю, ты открыл электролиз. В металле же электронов много, а сам ток является перемещением градиента а не самих электронов. Ну тоесть один ушел, на его место пришел другой, и так по цепочке. Пока электрона не было ли было слишком много, молекула была ионом, но в металле это не влияет на химическую связь. В жидкостях же ионы и так существуют и постоянно меняются электронами, ток лишь немного нарушает равновесие, создавая градиент потенциалов.
Аноним 01/02/18 Чтв 11:37:43  429980
>>429970
а антиматерию ты откуда брать будешь?
Аноним 01/02/18 Чтв 12:23:12  429984
Сап, кто читал линк (ТРАКТАТ О ЛЮБВИ http://ethology.ru/library/?id=12 )?
Я не особо умный, не хотелось бы забивать голову чьими-то фантазиями. Первые пять минут чтения, бреди не выявили, но написано каким-то двачерским языком. Можно читать?
Аноним 01/02/18 Чтв 12:45:57  429985
>>429956
>>429978
Спасибо, мне уже легче, с этим как-нибудь разберемся, всё, последний тупой вопрос и отстану: нам эта училка по химии ещё задвигала что вполне возможно что Солнечная система изначально была атомом, Солнце типа ядро, а планеты электроны, а потом обросло тёмной материей или чем-то таким, насколько это действительно может быть?
Аноним 01/02/18 Чтв 12:51:52  429986
>>429985
нинасколько, меняй школу. Она поехавшая.
Аноним 01/02/18 Чтв 14:11:24  429990
>>429985
> а планеты электроны
Бля, лол. Вообще в "квантовой эзотерике философии" возхможно наверное всё, что можно вообразить.
Аноним 01/02/18 Чтв 14:24:59  429992
>>429990
>Бля, лол. Вообще в "квантовой эзотерике философии" возхможно наверное всё, что можно вообразить.
Любая хрень нарушающая законы физики там внезапно получает нулевую амплитуду вероятности.
Аноним 01/02/18 Чтв 14:39:50  429994
>>429992
А что там не состыкуется? Это же кварк-глюоная плазма в условиях "экзотических" взаимодействий? Или что-то другое всё-таки?
Аноним 01/02/18 Чтв 14:47:53  429995
>>429992
>Любая хрень нарушающая законы физики
Константы могут быть совешенно разными, в разных вселенных.
Аноним 01/02/18 Чтв 16:25:25  429997
>>429673
Бред какойто.
Аноним 01/02/18 Чтв 16:38:16  429998
>>429953
>училка по химии ещё задвигала что вполне возможно что Солнечная система изначально была атомом, Солнце типа ядро, а планеты электроны
Скорей всего она проводила аналогию чтоб тебе было понятней что собой представляет атом. Хотя у меня была похожая идея - что сходство атома с солнечной системой не случайно и Вселенная это такой фрактал, но это было в поезде и после коньяка, так что эту теорию еще нужно доказать.
А вообще, училки по химии это пиздец, я до сих пор помню как моя утверждала что нагрев может дать только химическая реакция, и когда я поправил ее привев в пример термоэлектронную эмиссию, да подъебала "что, выучил умные слова и теперь крутой?".
Из-за подобной хуйни я сука до сих пор не монимаю химию, хоть и ВУЗ уже окончил.
Аноним 01/02/18 Чтв 16:48:55  429999
>>429985
училка жертва беспощадного научпопа? Но скорее всего просто не але не в своей области
Аноним 01/02/18 Чтв 17:02:22  430000
>>429247 (OP)
Методом сдвига точки сборки для ухода в третье внимание.
Аноним 01/02/18 Чтв 19:48:47  430009
Насколько теория относительности доказана? Ну, релятивистское замедление времени к примеру. Подразумевается ли оно в альтернативных теориях гравитации?
Аноним 01/02/18 Чтв 19:51:44  430010
>>429998
>нагрев может дать только химическая реакция
А какая химическая реакция происходит при трении двух поверхностей друг о друга?
Аноним 01/02/18 Чтв 20:29:28  430015
Есть два объяекта, оба движутся со скоростью 0.6с навстречу друг другу. Разве относительно друг друга их скорость не превысит скорость света?
Аноним 01/02/18 Чтв 20:32:14  430016
>>430015
А епт, все понял. Так как время для обоих объектов растягивается, то относительно друг друга они движутся медленее.
Аноним 01/02/18 Чтв 20:36:57  430017
>>430009
В тех случаях, для которых она справедлива, практически на сто процентов.
Аноним 01/02/18 Чтв 20:47:50  430018
>>430016
А если один объект движется почти со скоростью света и толкает другой объект в направлении движения?
Аноним 01/02/18 Чтв 21:09:54  430020
>>430018
Чем больше скорость. Тем сложнее толкать.
Аноним 01/02/18 Чтв 22:40:59  430024
>>429961
Ну блять, ну! Так вышло, вообще-то. Я бы и не поступал долбоёбом, если бы не обстоятельства.
Аноним 02/02/18 Птн 01:18:05  430025
>>429247 (OP)
правда что эволюция это абсолютно случайный процесс? Или она просчитывается теорией случайных процессов из математики?
Аноним 02/02/18 Птн 01:27:59  430026
>>429984
нет. Поведение человека слишком сложное и не укладываеться только в этологию.
Наверни курс соц биология человека на ютубе. Там на всех уровнях: эволюционном, генетическом, нейробиологическом и т. д. объясняеться.
Аноним 02/02/18 Птн 02:04:34  430027
>>429984
>написано каким-то двачерским языком
Лол, это скорее "двачерский" пошел из этого трактата. Ты реально ньюфаг или просто толстишь?
>>430026
Любое сложное можно упростить для понимания. В том трактате затрагиваются в более-менее понятной форме важные для простых людей вопросы, ты же предлагаешь зарыться в предметы с потерей важной инфы среди всяких определений, лекций, а в случае с Ютубом - еще и популизма.
Аноним 02/02/18 Птн 02:06:06  430028
>>430020
А как же относительность? Для фотона я и так двигаюсь со скоростью света, но могу передвигать предметы в любом направлении.
Аноним 02/02/18 Птн 02:47:24  430029
>>430028
>Для фотона
Такого понятия не существует. Фотон движется со скоростью света.
Аноним 02/02/18 Птн 03:38:54  430030
Пацаны, подскажите название самого крутого начального, вводного учебника по социологии на русском языке.
Аноним 02/02/18 Птн 08:52:16  430034
hydrogen.jpg (24Кб, 365x200)
Что будет с человеком, если поставлять ему будут доступны (т.е. из них будет состоять подаваемая ему еда, вода и воздух) только тяжелые стабильные изотопы основных элементов (которые в обычной среде практически отсутствуют), из которых состоит тело, и это будет продолжатся, пока подавляющее большинство атомов в его теле не будет заменено. Поэлементно дейтерий (0,01% в естественной среде), кислород-18 (0,2%), углерод-13 (1,1%), азот-15 (0,4%), кальций-44 (2%).
По массе уже прикинул, человек, который сейчас весит 75кг, будет весить чуть более 90 (при идентичном объеме и комплекции), но какие еще будут последствия?
Аноним 02/02/18 Птн 09:49:27  430037
>>430027
>>написано каким-то двачерским языком
Я имел ввиду, словечки эдакие, нарочито усложнённо описывается то, что можно было сказать проще (защита от сельдей, лол?), гуманитарии троллируются чуть ли не в первом абзаце.
Понятно, что описание со стороны этологии, не даст полной картины, но приблизит к пониманию. Я спрашивал, не начнется ли там утверждений, о том, что у женщины мозг как у утки и тд?
Аноним 02/02/18 Птн 09:58:45  430038
>>430034
Химические и водородные связи могут отличаться, а значит реакции могут протекать по-другому.
Так, напирмер:
Тяжёлая вода токсична лишь в слабой степени, химические реакции в её среде проходят несколько медленнее по сравнению с обычной водой,
водородные связи с участием дейтерия несколько сильнее обычных.
Эксперименты над млекопитающими (мыши, крысы, собаки) показали, что замещение 25 % водорода в тканях дейтерием приводит к стерильности,
иногда необратимой. Более высокие концентрации приводят к быстрой гибели животного;
так, млекопитающие, которые пили тяжёлую воду в течение недели, погибли, когда половина воды в их теле была дейтерирована;
рыбы и беспозвоночные погибают лишь при 90 % дейтерировании воды в теле.
Простейшие способны адаптироваться к 70 % раствору тяжёлой воды, а водоросли и бактерии способны жить даже в чистой тяжёлой воде.
Человек может без видимого вреда для здоровья выпить несколько стаканов тяжёлой воды, весь дейтерий будет выведен из организма через несколько дней.
Аноним 02/02/18 Птн 10:28:26  430041
Сап, вот вы пишете постоянно, е=мс2, а в чем получается это е? В кулонах? Как посчитать, сколько аккумуляторов я могу зарядить рублевой монеткой если гипотетически выковыряю из неё 100%
энергии
?
Аноним 02/02/18 Птн 10:31:32  430042
>>430041
https://ru.wikipedia.org/wiki/Джоуль#Перевод в другие единицы
Аноним 02/02/18 Птн 10:32:16  430043
>>430042
https://ru.wikipedia.org/wiki/Джоуль#.D0.9F.D0.B5.D1.80.D0.B5.D0.B2.D0.BE.D0.B4_.D0.B2_.D0.B4.D1.80.D1.83.D0.B3.D0.B8.D0.B5_.D0.B5.D0.B4.D0.B8.D0.BD.D0.B8.D1.86.D1.8B
фикс
Аноним 02/02/18 Птн 11:09:18  430046
>>430042
>>430043
Спасибо, я могу сейчас полную чушь нести, поправь или обоссы:
Что бы ответить на мой вопрос выше (сколько акумов можно зарядить монеткой), мне нужно массу перемножить на сс, полученные джоули(?) я могу перевести в ватты, далее получившееся я делю на 12 вольт, получаю хуиллиард ампер, разделив которые на объем аккумулятора (60А/ч), я получу количество аккумуляторов, которые я могу зарядить монеткой?
Я просто в этом нихуя не понимаю, услышал об этом утром, пока на работу собирался, в рандомном видосе про черные дыры.
Аноним 02/02/18 Птн 11:35:08  430047
>>430037
Если этот тот самый трактат что я читал когда-то, нет, не начнется. Там описываются человеческие отношения "снаружи", как их описывают для других животных.
Аноним 02/02/18 Птн 11:36:08  430048
>>430029
>Фотон движется со скоростью света.
Ну так, а если фотон принять за точку отсчета, выходит все остальное движется со скоростью света, а фотон неподвижен.
Аноним 02/02/18 Птн 11:38:52  430049
>>430048
Фотон нельзя принять за точку отсчета потому что фотон почти всегда движется со скоростью света. Смирись.
Аноним 02/02/18 Птн 11:39:34  430050
>>430038
> химические реакции в её среде проходят несколько медленнее по сравнению с обычной водой
А органические соединения получаются те же? Значит ли это что человек будет жить дольше т.к. реакции проходят медленней?
Аноним 02/02/18 Птн 11:42:15  430052
>>430049
И что? Движение(почти) со скоростью света не дает принять объект за точку отсчета?
А как же относительность тогда? Если объект с околосветовой скоростью по каким-то причинам перестает быть материальной точкой, задача о двух телах становится бессмысленной.
Аноним 02/02/18 Птн 11:47:59  430053
>>430052
>Движение(почти) со скоростью света не дает принять объект за точку отсчета?
Со скоростью света двигаются только безмассовые частички. При этом, безмассовые частички могут только двигаться со сокростью света и больше никак опустим экзотический вариант когда фотон можно якобы остановить, от него у мамкиных физиков срака рвется.

С (почти) и околосветовой скоростью проблем нет.
Аноним 02/02/18 Птн 11:54:07  430055
>>430053
Ок, фотон не может остановиться, но его можно замедлить, в то же время можно разогнать что-то другое с массой. Таким образом можно создать ситуацию, когда фотон относительно этого чего-то неподвижен.
Ладно, хуй с ней с остановкой, но почему точка отсчета фотона не относительна? Просто дело в массе?
Аноним 02/02/18 Птн 11:56:55  430056
>>430055
>его можно замедлить, в то же время можно разогнать что-то другое с массой
>создать ситуацию, когда фотон относительно этого чего-то неподвижен.
Удачи, чо. Не забудь учебник сначала почитать, а то обидно будет потом.
>точка отсчета фотона
Ее не существует.
Аноним 02/02/18 Птн 13:12:23  430058
>>430048
Можно сказать это да, сказать можно много чего, что угодно, только смысл.
Аноним 02/02/18 Птн 13:40:48  430059
>>430048
>выходит все остальное движется со скоростью света, а фотон неподвижен.
ну да, и что? для него время останавливается
Аноним 02/02/18 Птн 13:47:41  430061
>>430055
>Таким образом можно создать ситуацию, когда фотон относительно этого чего-то неподвижен.
можно создать ситуацию, когда относительно стороннего наблюдателя частица с массой будет двигаться быстрее фотонов
Аноним 02/02/18 Птн 14:34:22  430062
>>430046
Во-первых, ёмкость аккумуляторов измеряется в кулонах, и это не в A/ч, а в A·ч. Так как в часу 3600 секунд, то это 3600 А·с
https://www.unitjuggler.com/перевод-electriccharge-из-Ah-в-As.html
Во-вторых, там на википедии можешь видеть следующее: 1 Дж = 1 кг·м22= 1 Н·м = 1 Вт·с = Кл·В
Вт - это ватт. Вт = В·А, а м22 = (м/с)2 - квадрат скорости.
Таким образом, кг·(м/с)2 = В·А·с,
и здесь как-бы вырисовывается формула: mc^2 = U·q, где q - заряд аккумулятора.
Эта же формула, через E указана здесь: https://ru.wikipedia.org/wiki/Ампер-час E = q · U, но вместо E - я подставил mc2.
Вольты у тебя есть. Ампер-часы тоже. Как перевести ампер-часы в ампер-секунды - я показал.
Можешь ещё Джоули эти свои как ватт ·секунды считать, т. е. через мощность.
Из формулы mc^2 = U·q получается следующее: q = mc^2/U,
и поскольку скорость света - константа (299 792 458 м/с), и квадрат её - константа (89875517873681764 м22),
и напряжение у тебя постоянное - 12 вольт, то зависимость заряда от массы - зависит от одной лишь константы:
89875517873681764 м22 / 12 В = 7489626489473480.(3) м22
Т. е. заряд растёт в зависимости от массы и этой константы: q = m·7489626489473480.(3) (м22/В)

Действительно, вполне возможно было бы такой заряд и получить, если все кварки внутри адронов в материи монетки - заставить двигаться,
в направлении противоположном их кулоновскому притяжению. Но сделать это можно было бы в теории и, только в случае если монетка является биткоином.
Аноним 02/02/18 Птн 14:40:33  430064
>>430062
>только в случае если монетка является биткоином
>мне нужно массу перемножить на сс
Я-то думаю чё это у меня после твоей монетки - биткоин в уме крутится? Мимо из /cc
Аноним 02/02/18 Птн 14:44:38  430066
https://www.youtube.com/watch?v=zH459qgNimk&t=344s
Аноним 02/02/18 Птн 14:48:33  430067
>>430046
>Что бы ответить на мой вопрос выше (сколько акумов можно зарядить монеткой)
да, кстати, вот >>430062
и поскольку каждый аккум у тебя имеет вполне определённый заряд, то это q делишь на q аккумулятора - получаешь количество аккумуляторов.

>>430043>>430064-кун.
Аноним 02/02/18 Птн 15:02:37  430068
>>430059
Нет, это время окружающей его вселенной летит неебически бесконечно быстро.
Аноним 02/02/18 Птн 15:04:45  430069
>>430068
И тут возникает вопрос: Какую скорость относительно этой вселенной "ощущает" сам фотон?
Аноним 02/02/18 Птн 15:33:23  430071
>>430026
Кек, если ты имеешь ввиду курс лекций Сапольского, то его просмотр закончится как раз на лекциях по этологии, потому что далее идут генетика/эпигенетика, в которые вкатиться с дивана не получиться.
>>429984
Читай, всё-равно лишним не будет. Только учти, что здесь много обобщённой информации, поэтому после прочтения не надо кричать, что все бабы шлюхи, ебущиеся с ваньками ерохинами. Как тебе уже сказали, на поведение человека влияет очень много факторов и без закапывания в кучу книг по эволюции, генетике, эпигенетике и т.д. ты не увидишь полную картину и не сможешь осознать, что движет людьми на самых фундаментальных уровнях.
Аноним 03/02/18 Суб 09:39:32  430124
95.png (23Кб, 715x581)
>>429247 (OP)
анон, скажи а критическая масса деления от чего нибудь зависит? можно ли снизить её при помощи высокой например температуры и давления? можно ли создав маленькую точку с очень высокой температурой и давлением того же урана, снизить его критическую массу до грамма?
Аноним 03/02/18 Суб 10:45:41  430127
>>430124
Наверняка зависит от температуры и давления, вроде в реакторах реакция проходит при большом нагреве, еще минимизирует к. массу наличие отражателей, но какой минимум я хз, погугли.
Аноним 03/02/18 Суб 11:19:45  430133
>>430124
Хуй знает, в гугле пишут, что зависит от чистоты матерьяла, формы и отражателей.
Бля, роскомнадзор зашугал, уже очкую такое гуглить.
Аноним 03/02/18 Суб 11:20:20  430134
>>430133
>матерьяла
Ну вы понели)
Аноним 03/02/18 Суб 11:28:45  430136
>>430124
При помощи нейтронных отражателей можно, но может ебануть.
Аноним 03/02/18 Суб 11:32:13  430138
>>430071
>>430067
>>430062
Спасибо, но это уже слишком сложна для меня. Про аккумуляторы это я к примеру сказал, интересно практически понять, сколько энергии в этой ебанной монетке.
Кстати в том видосе говорилось про эффективность извлечения энергии из вещества, всякие и ядерные реакции позволяют получить совсем малый % энергии, а самый эффективный способ, бросить вещество в ЧД, и при падении оно разгонится до около СС, и в виде излечения отдаст нихуевый такой процент энергии. Так сказали в том видосе.
Аноним 03/02/18 Суб 11:34:15  430139
>>429247 (OP)
Было уже. Некто Гилберт Госсейн (персонаж) в совершенстве овладел способностью создавать квазивиртуальную картину отдельных точек пространства/времени в собственном сознании идентичную оным в фактически существующем континиуме, благодаря чему телепортировался к вышеозначенным.
Аноним 03/02/18 Суб 11:40:23  430140
>>430124
Ути-пути личинка дезинтегратора вселенной путём нового/первого big-bang. Если считать теорию big-bang верной, то само собой пути есть. Сами-то вы какой концепцции происхождения современного континиума приберживаетесь коли не секрет?
Аноним 03/02/18 Суб 11:58:23  430144
>>430052
Свет одновременно и волна и частица, это раз.
Разные типы колебаний распространяются с разной скоростью. К примеру звук пердежа и взрыв сверхновой.
Коли разные типы колебаний распространяются/перемещаются с разной скоростью накой вы прицепились к свету? Поищите что-нибудь новое, пошустрее. Выполнить, само-собой не столь просто как написать, но писать обычно проще в подавляющем большинстве случаев.
Алсо если найти тип колебаний распространяющихся быстрее света пропорционально настолько же, как сыет быстрее звука, что вам помешает измыслить сперва математичечкие, а впоследствии и практично-физические модели движения с последующим их переводом в область промышленного производства?
Аноним 03/02/18 Суб 12:19:35  430149
>>430050
В данном случае нам важна субъективная, а не объективная длительность бытия индивидуума. Она же остаётся неизменной при любой скорости движения.
Путешествие же субъекта к иным звёздным системам суть действие сомнительное с точки зрения практического смысла, поскольку неизвестны по причине отсутствия данные, подтверждающие однородность законов физических полей, как-то атомарные связи, гравитация, волновые скорости атомарных конструкций в текущих средах.
Упрощая написанное - лежит стопка разнородных тканей текстильного производства (шелк, драп, сафьян) олицетворяющих различность свойств физического пространства. Кораблик первопроходцев находится в одном из слоёв. Вы действительно считаете что ему будет ОК в пространстве иного типа? Непрактичные рассуждения, являющие собой однако возможность тренировки мышления.
И вот вам в таком случае задачка:
Две звезды движутся навстречу друг другу со скоростями в 1/3 и 3/2 скорости света в вакууме, т.е. сближаются со световой скоростью. Вы уверены что фотон, отрывающийся от поверхности одного из указанных светил, будет иметь скорость относительно неподвижного пространства равную 1? А если он движется в сторону движения звезды? А если в противоположную? Это всё бабка надвое сказала какая у него будет фактическая скорость, поскольку для проведения корректного опыта необходимы замеры с несуольких точек, каждая из которых движется с отличной от нуля и от точки-оппонкнта скоростью!
Аноним 03/02/18 Суб 12:32:20  430150
Можно ли древесину превратить в жидкость? Чтоб химически это было то же самое
Аноним 03/02/18 Суб 12:32:56  430151
>>430140
>Сами-то вы какой концепцции происхождения современного континиума приберживаетесь коли не секрет?
Я не хочу собирать дезинтегратор либо ядерную бомбу для того чтобы захватить мир. Не моя тема. А вот сделать аналог плазменного ракетного двигателя, где рабочим телом будет какой нибудь нестабильный изотоп, склонный к делению и использовать его как топливо в этом двигателе чтобы замутить супер-крутой ракетный двигатель вот это мне ближе.
>>430124
Аноним 03/02/18 Суб 12:35:50  430152
>>430150
если её в герметичной таре нагреть чтобы исключить горение, то я думаю постепенно разные её части начнут плавиться, но при остывании обратно древесиной не станут. Думаю вода и углерод разделятся изза температуры. После остывания будет гыча из воды и угля
Аноним 03/02/18 Суб 12:37:42  430153
>>430140
иными словами взять тот же токмак для реакции синтеза, но запихать в него например нестабильный цезий, нагретый до состояния газа, дальше он при теоритической низкой критической массе начинает деление, давая охуенную реактивную струю. тоесть направленный мини-ядерный взрыв в качестве реактивной тяги
Аноним 03/02/18 Суб 12:58:30  430156
>>430144
Если попробуешь что-то разогнать выше сс, вселенная схлопнется нахуй в сингулярность.
Аноним 03/02/18 Суб 15:12:11  430168
>>430025
Бамп
Аноним 03/02/18 Суб 17:52:43  430187
Объясните новичку, как происходит превращение вещества в плазму?
Аноним 03/02/18 Суб 23:40:51  430209
чёрные дыры взрываются сразу же после их образования, но т.к. время в их пределах движется для стороннего наблюдателя бесконечно медленно, то для того же стороннего наблюдателя они не взрываются вообще?
Аноним 04/02/18 Вск 00:06:45  430210
>>430187
Нагревом и/или ионизацией. Так же как жидкость превращается в газ.
Аноним 04/02/18 Вск 11:02:30  430232
>>430153
>давая охуенную реактивнуюструю
Мы так же в детстве соревновались.
В любом случае для меня, давно и хорошо знающего человечество, конечный результат подобных разработок ощущается далёким от пацифизма. Да и расход топлива присутствует.
Так же как паровые телеги на угле заменили лошадей, а их заменили двс, которых уже потихоньку заменяют аккумуляторные изделия, так же стадии смены типов двигателей должно пройти и ракетостроение. А реактивная струя - детство человечества, от которого оно (слава богу, цк кпсс и научно-промышленному прогрессу) в скором времени отойдет.
Что будет выкидывать летательные аппараты в ближний космос? Возможно антиграв-двигатель, принцип которого изучали в 8х классах на уроке физики, возможно сверхёмкие аккумуляторы в паре с ним. Возможно это будет орбитальный лифт. В телепортацию в ближайшие лет 800 я не верю, по крайней мере крупных объектов, атомы-то уже телепортируют.
Аноним 04/02/18 Вск 11:36:05  430233
>>430232
>двс, которых уже потихоньку заменяют аккумуляторные изделия
Дальше не читал фантазёра.
Аноним 04/02/18 Вск 16:21:27  430281
>>430210
Я про процесс превращения спрашиваю, как это происходит.
Аноним 04/02/18 Вск 16:22:18  430282
>>430210
Газы все разные, у них разные свойства, показатели итд
Плазмы то же все разные?
Аноним 04/02/18 Вск 16:49:07  430283
uuiui.jpg (42Кб, 400x426)
>>430282
бля, интерксный вопрос,должны быть одинаковы..
Аноним 04/02/18 Вск 17:28:12  430287
>>430283
Так атомы же там разные.
Аноним 04/02/18 Вск 17:38:53  430288
ееееккк5.png (62Кб, 272x274)
>>430287
а ионизация-то одинаковая))
Аноним 04/02/18 Вск 20:32:14  430304
>>430283
атомы разные, значит, и плазма разная
Аноним 04/02/18 Вск 20:39:35  430305
3у345.png (94Кб, 441x462)
>>430304
атомы из одной хуйни сделаны))
Аноним 04/02/18 Вск 21:19:22  430307
Тупой вопрос: чем обусловлен гироскопический эффект на фундаментальном уровне?
Аноним 04/02/18 Вск 21:32:49  430308
ллг765.png (151Кб, 706x636)
>>430307
вопрос изначально неправильный
Аноним 04/02/18 Вск 22:41:15  430315
>>430305
>атомы из одной хуйни сделаны))
Т.е. при образовании плазмы атомы распадаются на элементарные частицы?
Ну бред же
Аноним 04/02/18 Вск 22:52:25  430318
>>430069
Никакую. Он "умирает" в тот же момент что и "рождается".
в одном видосе видел

Хм, вспонил тот видос и подумалось-скорость его же зависит от вещества через которое он проходит.
А на земедление времени влияет вроде только абсолютная скорость, а не то, какая она для света в определённом веществе.
Аноним 05/02/18 Пнд 00:08:32  430328
А что если плазму мгновенно замарозить?
Аноним 05/02/18 Пнд 00:16:32  430330
>>430315
А я подозреваю, что во Вселенной двух одинаковых атомов любого вещества не сыскать, так, между нами...
Аноним 05/02/18 Пнд 12:10:56  430360
>>430233
>кококо
http://russian.people.com.cn/31518/8609542.html
Линканул тебе за щеку
Аноним 05/02/18 Пнд 12:17:19  430365
>>430233
Полчаса зарядки на 650км пути, 5 секунд до сотки.
https://korrespondent.net/lifestyle/motors/3907556-elektrohruzovyk-ot-Tesla-predstavlen-ofytsyalno
Итого линканул за обе щеки деревенщине
Аноним 05/02/18 Пнд 12:18:39  430366
>>430330
Почему? В слитке палладия рядом лежат миллиарды одинаковых атомов. Ну может быть положение электронов на орбите не совсем совпадает, но не придераться же к этому.
Аноним 05/02/18 Пнд 12:19:47  430368
>>430365
Представляю как эта пседозелёная технология загрязнит природу использованными аккумуляторами.
Аноним 05/02/18 Пнд 15:10:42  430384
>>430050
У тебя реакции где рвутся связи водород-элемент начинают медленнее идти раз в 5-6, а многие другие реакции не меняют скорости. Ты уверен, что хочешь узнать к чему это всё приведет?
Аноним 05/02/18 Пнд 15:14:20  430386
>>430152
После нагревания будет гыча из синтезгаза.
Аноним 05/02/18 Пнд 15:15:27  430387
>>430330
Если бы одинаковых атомов нельзя было сыскать. То запреты Паули бы не выполнялись...
Аноним 05/02/18 Пнд 15:26:41  430393
>>429247 (OP)
Хочу вкатиться в квантмех. Какой нужен бэкграунд? Если я изучаю вышмат, нужен полный курс или какие-то темы можно пропускать?
Аноним 05/02/18 Пнд 15:51:14  430394
>>430393
Линал и немного ТФКП
Аноним 05/02/18 Пнд 18:04:01  430408
>>430394
Благодарю. Раз уж тупых вопросов тред. Законспектил учебник - элементы линал, элементы вектал и порешал всякое. ТФКП в самом конце учебника по вышмату. Логично, что перед ТФКП нужно изучить комплексные числа, наверное что-то ещё нужно?
Аноним 05/02/18 Пнд 18:26:09  430411
Можно ли начать излучать в радиодиапазоне или начать светиться, если очень быстро начать обмахиваться веером?
Аноним 05/02/18 Пнд 20:18:22  430429
Реально ли высосать атмосферу через трубу в космос?
Аноним 05/02/18 Пнд 20:25:51  430430
Что скажите по поводу эффекта наблюдателя, господа?
Аноним 05/02/18 Пнд 20:48:31  430432
>>430430
Измерение производит изменение в результате декогеренции.
Аноним 05/02/18 Пнд 21:30:22  430438
мдм.jpg (60Кб, 797x446)
>>430432
Хм, надо бы это обдумать...
Аноним 06/02/18 Втр 01:02:13  430453
>>430328
Будет газ, жидкие аэрозоли или взвешенные частички твердого тела, в зависимости от того как сильно заморозить
Аноним 06/02/18 Втр 17:56:10  430482
Почему ток в катушке индуктивности продолжает течь в прямом направлении после размыкания цепи? Насколько я понял процесс, ток порождает мп, которое переориентирует домены в магнитопроводе, это не мгновенный процесс. После отключения тока домены возвращаются в исходное состояние, создавая изменение магнитного поля и порождают ток в противоположном направлении. Но почему при размыкании цепи(или увеличении последовательно включенного сопротивления) ток сначала течет в том же направлении? Ведь переориентация доменов должна сразу прекращаться после отключения тока и сразу же начинать идти в обратном направлении.
Аноним 06/02/18 Втр 19:08:34  430488
>>430482
Ток самоиндукции при размыкании катушки возникает вследствие возбуждения ЭДС самоиндукции,
а она возбуждается по закону электромагнитной индукции при изменении магнитного потока.
Гугли по выделенным ключевым словам.
Аноним 06/02/18 Втр 19:30:31  430490
blob (46Кб, 460x502)
sol2[1].jpg (27Кб, 343x256)
>>430488
Сердечник вообще не рассматривай, только катушку.

Магнитное поле в проводнике возбуждается по закону Био — Савара — Лапласа и вроде-бы по закону Ампера - Максвелла,
и имеет направление, определённое правилом буравчка, пик1.
Магнитное поле в катушке соленоида (прямая катушка с витками в одну сторону) - представляет из себя сложение магнитных полей проводников этих витков.
Пик2.
Когда ток падает, магнитное поле убывает, т. е. изменяется.
А при изменении магнитного поля (убывании) - индуцируется ЭДС самоиндукции.
https://www.youtube.com/watch?v=KtMGOgxnp9c на 14:19 показано направление ЭДС самоиндукции при ослаблении (убывании) магнитного поля, а затем при его усилении.
И вообще, посмотри можешь посмотреть весь фильм и тут ещё порыться: https://www.youtube.com/playlist?list=PL69_OWVXSzBILR4cZKFk8b4VuZLa079wt
про магнетизм, электрическое поле и вот это вот всё.
Аноним 06/02/18 Втр 20:03:19  430494
>>430366
бред, иди в школу доучивайся.
Что ты за ахинею несёшь?
>>430387
ещё один неуч, вообще сам соображать не умеет, что ты сморозил за херню.

Всё именно так, как я сказал, во всей Вселенной не найти двух одинаковых частиц.
Аноним 06/02/18 Втр 20:03:56  430495
Ну так что с плазмой? Разные атомы - разная плазма?
Аноним 06/02/18 Втр 20:13:19  430498
изображение.png (16Кб, 746x1452)
>>430488
>>430490
Посмотрел, вроде как я это и знал, но всеравно чет не понятно. Вот посмотри на пикрил:
1-я схема: проводник с током и мп, им порожденное.
2-я схема: отключение тока от проводника, мп поле меняется в другую сторону.

Легенда графиков:
Красный - напряжение на источнике
Оранжевый - ток в проводнике, как он есть: сначала продолжение затем только в противоположную сторону.
Зеленый - ток в проводнике каким он должен быть: после отключения тока сразу в противоположном направлении.
Голубой - вектор намагниченности
Фиолетовый - производная, этого вектора, которая показывает направление изменения мп и соответственно тока в проводнике.

Собсно тут 2 вывода:
1. Если я все правильно нарисовал, почему ток течет сначала в том же направлении а только в противоположном.
2. Если я ошибся и нижний график должен быть наоборот(сначала мп порождает ток в противоположном направлении а затем в прямом), то почему ток после прямого направления течет еще и в противоположном(оранжевый график)?
Аноним 06/02/18 Втр 20:27:16  430502
c23df0a140cb5f2[...].jpg (385Кб, 849x1200)
>>430430

1)
Почему Квантовая Физика - Бред?

Смотрим ролик:
https://www.youtube.com/watch?v=LSwCCtvbSdg


А что же происходит на самом деле?

Электрон состоит из:
A-составляющей и B-составляющей.

Электроны проходят через две щели, возникает интерференционная картина, из-за волновой А-составляющей электронов.

Одиночный электрон проходит через две щели сразу и интерферирует сам с собой из-за А-составляющей, но при достижении поверхности, мы видим его точечную B-составляющую. "Невидимая", в данном случае, A-составляющая тянет за собой видимую B-составляющую и поэтому мы получаем интерференционную картину при большом количестве одиночных электронов.

Когда мы начинаем смотреть, через какую именно щель одиночный электрон проходит "на самом деле", мы физически вмешиваемся в его движение (например, испуская/впуская фотоны, которые сталкиваются с электроном), вследствие чего пропадает интерференционная картина, мы видим B-составляющую, а воздействие A-составляющей нейтрализуется. И мы приходим к заблуждению, что электрон - частица, когда мы подглядываем за ним, и волна, когда не подглядываем.


Чтобы доказать мою теорию, нужно провести ряд экспериментов.

На самом деле, электрон может состоять не из двух, а из тысяч мелких составляющих (или больше), устройство его системы может оказаться крайне сложным, но это в данном случае не имеет никакого значения.

Квантовая физика - заблуждения и бред, и всё из-за непонимания устройства частиц.

2)
Формула неверна:
Δx + Δp = h/2
из-за того, что это выдерка из общей системы и общая картина пока не видна (только идиот станет сомневаться).

Верная формула:
Δx + Δp ?A? = h/2 ?B?
как-то так.
Кроме того, возникают вопросы насчёт знака "=". Там могут быть: "≈", "->", "<-" или даже ничего из этого.


Я про, вы - нет.
Аноним 06/02/18 Втр 21:59:35  430509
>>430502
>Формула неверна:
>Δx + Δp = h/2
Конечно не верна :-)

Аноним 06/02/18 Втр 22:38:53  430516
>>430502
ловите наркомана
Аноним 06/02/18 Втр 22:41:24  430518
>>430494
>Всё именно так, как яскозал
словоблуд плиз
Аноним 06/02/18 Втр 23:52:50  430523
222-1-44[1].png (7Кб, 940x246)
li03015[1].jpg (7Кб, 395x179)
>>430498
>Оранжевый - ток в проводнике, как он есть: сначала продолжение затем только в противоположную сторону.
>Зеленый - ток в проводнике каким он должен быть: после отключения тока сразу в противоположном направлении.
Там в RL-цепях (резистор и катушка) ещё есть отставание тока от напряжения - фаза тока в индуктивности отстает от фазы напряжения на 90°.
Поэтому у тебя и здвиг зелёной линии и оранжевой.

>Если я все правильно нарисовал, почему ток течет сначала в том же направлении а только в противоположном.
Самоиндукция же. Пик1. Направление тока самоиндукции определяется по правилу Ленца:
ток самоиндукции всегда направлен так, что он противодействует изменению основного тока.
Если основной ток возрастает, то ток самоиндукции направлен против направления основного тока,
если уменьшается, то направления основного тока и тока самоиндукции совпадают.
http://www.physbook.ru/index.php/SA._Самоиндукция

>почему ток после прямого направления течет еще и в противоположном(оранжевый график)
Может у тебя там что-то перемагничивается? Хотя не должно, ведь при размыкании ток самоиндукции течёт
по причине ослабления магнитного поля. Там, вроде, не должно быть противоположно направленного магнитного поля.
Аноним 07/02/18 Срд 00:38:02  430526
>>429565
>Так и смотри видео на ютубчике про Тибет - будешь в тибете по своей логике,
>но это не называется телепортацией, ты просто подменяешь понятия.
Тут недавно, пол-года назад, где-то в этом разделе - был тред про самосожжение монаха,
а вот же он: https://2ch.hk/sci/arch/2017-09-24/res/408592.html
и я помню описательное моделирование этой ситуации - в Тибете, лет 20 назад.
Вот это телепортация! И не моя это вовсе логика. Тут нечто более глобальное.
Меня просто притащили туда и показали это, что тут будет так, и это правда.

И по моей логике - не обязательно было вообще находиться там, чтобы узреть сие,
причём также качественно, как всё-же получилось приподнести это - там.

>>429746
Годно, но... Вместо дрона мог бы быть универсальный человекоподобный андроид.

>>429764
>ты не сможешь попить водицы из горы Фуджи, отправив дрона или еще что-то
Ну, можно было бы привинтить на дрон нечто вроде глотки и рук,
а потом пойти просто воды попить, и ассоциативно домоделировать в сознании все тонкости черпания воды,
и заглатывания её, с её журчанием, и вкусом минералов. Химический состав можно было бы взять с сенсоров.

>>429766
>Если ты хочешь попить водицы, чтобы провести её химический анализ - сделай такой сенсор для квадрокоптера.
По-моему, он имел в виду несовместимость эстетических потребностей человеческого мозга с функциональностью квадрокоптера.

>Гипотетическиймозг вполне может лежать в банке в лаборотории, а сенсоры ездить по Марсу.
Пропуская инфу от сенсоров с Марса через нервы, такой мозг наверняка испытывал бы тяжкую душевную боль.
Особенно после необратимых фиксаций неактуальных значений
с последующей структуризацией нейросетевых структур в результате всего этого,
и дальнейшей миелинизацией образующихся нервных волокон.
Поэтому, как вариант - ИНС в лабораторной банке, причём тождественная изначальному мозгу
с реализацией процесса становления сознания в ней.

>>429760
>Понятное дело, что если ты хочешь телепортироваться, чтобы избежать пиздеца в текущей точке пространства
>для своего обработчика информации - мозга, то такое не подходит.
Но подобная телепортация могла бы эффективно реорганизовывать причины процессов к нему ведущие,
причём без необходимости проведения какого-либо управления со стороны изначального мозга.
Допустим, мозг хочет телепортироваться, и у него не получилось из-за угрозы пиздеца в текущей точке пространства,
но квадрокоптер подхватывает желание телепортироваться и нивелирует причину пиздеца, с последующим обеспечением телепортации.
Но как? Квадрокоптер - там?!! А вот так - идёт синхросигнал любому другому квадрокоптеру поблизости, он летит и изучает инфу,
а возможно даже и влияет на среду вблизи причины негативного процесса, направленного на мозг, например нивелируя её.
А затем восстанавливается связь квадрокоптера с мозгом в состояние изначальной телепортации.
Что-то типа кооперативного гомеостаза различных телепортаторов - по ключевым параметрам
необходимым для функционирования самой телепортации.

Или такой вариант у мозга получилось телепортироваться, он с квадрокоптером связан, но долго мозг не может быть в этом состоянии,
но квадрокоптер подхватывает желание продолжить телепортацию и собирает инфу через другой квадрокоптер
о состоянии мозга и о состоянии среды вокруг мозга. Этот квадрокоптер распознаёт машинным зрением каким-нибудь,
"причину пиздеца у мозга в текущей точке пространства" и нивелирует её, гарантируя стабильное и устойчивое состояние телепортации.
Аноним 07/02/18 Срд 02:50:31  430539
>>430502
я согласен с тобой, ты мой герой, я хочу от тебя ребёнка.
Аноним 07/02/18 Срд 09:21:16  430549
>>430502
>Формула неверна:
>Δx + Δp = h/2
Какую-то формулу запхнул, непонятную. Если это принцип неопределённости Гейзенберга, то там знак умножения.
>Верная формула:
>Δx + Δp ?A? = h/2 ?B?
Что означает знак вопроса в этих формулах?
Аноним 07/02/18 Срд 13:30:06  430569
>>430549
>Что означает знак вопроса в этих формулах?
Не спеши, он еще не придумал.
Аноним 07/02/18 Срд 13:36:07  430570
>>429247 (OP)
ВНИМАНИЕ АНТИГРАВ
Берём сферическую систему Земля - Луна в вакууме. То есть кроме них больше ничего. Луна вращается вокруг Земли и не "падает" на неё, всё нормально... Постарайтесь представить себе это визуально я не силён в анимации
В первом случае Земля стоит на месте, Луна вращается вокруг неё. Но ту же самую систему можно представить по другому: Земля просто вращается, стоя на месте, Луна тоже стоит на месте - ничего не изменилось. Это та же самая система, те же самые векторы движения, но наблюдатель стоит на Луне. То есть относительно наблюдателя с Луны, Земля просто вращается, стоя на месте. Как такое возможно? Ведь что бы не упасть, объект должен вращаться ВОКРУГ другого.
Аноним 07/02/18 Срд 13:38:12  430572
>>430570
Земля и луна вращаются вокруг ообщего центра масс. Задача Кеплера.
Аноним 07/02/18 Срд 16:04:02  430588
Как-то услышал краем уха, что есть галактики, которые разлетаются быстрее скорости света и не придал значения. А теперь придаю. Что это такое? С одной стороны твердо и четко, что не можем достигнуть скорости света, а с другой такое.
Второе. Как мы их можем видеть если они летят быстрее скорости света? Мы же тоже, как бы от них разлетаемся. И вообще, может это мы разлетаемся со скоростью света, а думаем на другие галактики, как раньше думали, что это солнце вокруг нас вращается, а вышло наоборот.
Аноним 07/02/18 Срд 16:06:51  430589
>>430570
>Но ту же самую систему можно представить по другому: Земля просто вращается, стоя на месте, Луна тоже стоит на месте
Нит, нельзя, ибо система неинерциальная. Попробуй представить троллейбус неподвижным а планету движущуюся относительно него. Что тебя толкает на остановках?
Аноним 07/02/18 Срд 19:52:19  430607
>>430589
Но ведь теория относительности

Всмысле что толкает? Я же на месте стою
Аноним 07/02/18 Срд 20:11:01  430608
>>430588
Насколько я помню, на телескопе Хаббл как-то обнаружили далёкие галактики,
которые разбегались в разные стороны намного быстрее чем должны были бы.
По-моему там даже сверхсветовое движение засекли, и объяснили это ускоряющимся расширением
пространства Вселенной - за счёт темной материи и тёмной энергий.
Потом появились постоянная хаббла: https://ru.wikipedia.org/wiki/Постоянная_Хаббла
и объем Хаббла https://ru.wikipedia.org/wiki/Объём_Хаббла
На самом деле никакого сверхсветового движения не происходит, всё в пределах светового конуса,
а расстояние как-бы увеличивается за счёт расширения самого пространства.
Если на поверхности воздушного шарика нарисовать две точки,
и постепенно его надувать (пространство шарика расширяется), то расстояние между точками будет расти.
Если допустить, что точки эти движутся по поверхности шарика равномерно, не ускоренно, и отдаляясь друг от друга
(они разбегаются, как разлетаются галактики, не связанные особо гравитационным взаимодействием),
то при надувании шарика - скорость их разбегания будет увеличиваться, как будто бы они разбегаются ускоряясь.

Но поскольку далёкие галактики находятся на расстоянии в миллиарды световых лет,
то свет от них идущий сюда, показывает их состояние, которое было у них вот эти самые миллиарды лет назад...
И если речь идёт о скорости разбегания между
какой-нибудь галактикой EGSY8p7 (которая на расстоянии более 13 миллиардов световых лет от нас)
и галактикой UDFj-39546284 (эта галактика находится на расстоянии 13,3 млрд световых лет от Земли),
то скорость их разлета между собой закономерно увеличивается при наблюдении,
потому что пространство тогда расширялось относительно-интенсивно,
и эти галактики не очень поздно сформировались относительно нулевого времени,
поэтому и продолжали "ускоренно" разлетаться - вместе с расширяющимся пространством.
Что касается нулевого времени - согласно модели ΛCDM, возраст Вселенной составляет 13.799 ± 0.021 миллиардов лет,
т. е. до этого - было нулевое время, времени не было просто внутри сингулярности.
Аноним 07/02/18 Срд 21:31:39  430613
>>430607
Ну, ты стоишь на месте, когда троллейбус тормозит тебя немного сносит в направлении движения, а когда газует наоборот. Если за систему отсчета принять троллейбус, тебя сносить будет неизвестная сила.
Аноним 07/02/18 Срд 21:57:07  430615
яняша.jpg (385Кб, 849x1200)
>>430502

дx дp ?A? = h/2 ?B?

fixed

Дефолтная формула неполна, вот что имеется ввиду. И насчёт знака равенства вопросы есть.
Аноним 08/02/18 Чтв 08:42:05  430629
Блять наркоманы в треде
Аноним 08/02/18 Чтв 13:58:27  430637
Какие аксиомы и правила вывода нужно задать компьютеру, чтобы он смог вывести существование Тьюринг-невычислимых задач. А вернее можно ли это сделать вообще ? Автоматический вывод теорем уже норм вроде работает.
Аноним 08/02/18 Чтв 14:47:47  430639
>>430638
Т. е. ИИ на машине Тьюринга способен осознать существование не разрешимых для него задач ?
Аноним 08/02/18 Чтв 15:39:18  430642
Наберите в гугле мясовичок, не пожалеете!!!
Аноним 08/02/18 Чтв 16:15:03  430643
>>430640
Не зачем, просто мне интересно на сколько близки машина Тьюринга и мозг человека. Вот я задумался мы осознали существование Тьринг не вычислимых задач, а машина способная решать только Тьюринг вычислимые задачи сможет осознать наличие невычислимых ? Ты вроде сказал да, может, правильно я понял ?
Аноним 08/02/18 Чтв 17:05:01  430645
>>429670
если так рассуждать, то мы вернемся к тому вопросу, который и дал продолжение современной науке - если ты с фонариком залезешь на поезд, после поезд тронется со скоростью 72км/ч, то свет фонарика будет распространяться c+20м/с, а уравнения Максвелла такого финта ушами описать не могли, а попытки их допилить, чтобы подстроить под такую ситуацию, лажали. Например, если лететь вдогонку за светом с такой же скоростью — со скоростью света, — то этот свет по уравнениям Максвелла… исчезал. Если сократить пару сложных букаф, то можно перейти к постулату, что все физические процессы в инерциальных системах отсчёта протекают одинаково, и всё равно, неподвижна ли эта система или она находится в состоянии равномерного и прямолинейного движения. Вот и всё решение.
Аноним 08/02/18 Чтв 17:48:14  430649
яняша.jpg (385Кб, 849x1200)
>>430642
facefuck hard anal наберите то же не пожалеете, а может ещё больше не пожалеете.
Аноним 08/02/18 Чтв 19:07:26  430657
Не знаю куда с таким вопросом, напишут тут. Какие авторы и учебники лучше по биологии, интересует с самого начала. С создание плонет и солнечной системы до строение животного и человеческого организма.
Аноним 08/02/18 Чтв 19:13:49  430660
Вопрос уровня ребёнка 5 лет наверно. Где вокруг меня бактерии ? Они вообще всюду ? Вот на бумажных обоях, на лакированном столе они есть ? Или нужно обязательно чтобы от меня отлетел кусочек органики, которым они и будут питаться, а когда съедят то все передохнут ? Если есть бактерии, которые едят не оганические соединения, то почему они не съедают вообще всё вокруг ? Я прокалил железку в стерильных условихя, будут ли на ней бактерии, откуда возьмутся ?
Аноним 08/02/18 Чтв 20:03:38  430662
>>429247 (OP)
>Как можно использовать многомировую интерпретацию квантовой механики чтобы получить сверхспособности?
Высокоточное математическое моделирование всех альтернативных вариантов развития событий,
при осуществлении различных цепочек выборов во всех мирах сразу - за счёт квантового параллелизма в квантовых компьютерах,
с целью поиска наилучшего варианта последовательности выбора в реальном мире, и своевременного исполнения этой последовательности.
Т. е. если изменение одного параметра в одном мире может изменить какой-либо параметр в другом мире,
это изменение может и должно быть подобрано наилучшим образом ДЛЯ ВСЕХ МИРОВ сразу.
Аноним 08/02/18 Чтв 20:45:17  430664
>>429850
бамп, может реально посоветуйте или хотя бы по хардкору поясните за психогенетику.
Аноним 09/02/18 Птн 05:58:58  430696
>>429247 (OP)
Сап, двач. Где моя зарядка от ноутбука?
Аноним 09/02/18 Птн 14:52:56  430718
>>429330
Net
Аноним 09/02/18 Птн 14:58:42  430719
>>430660
>Где вокруг меня бактерии ?
Вокруг тебя.
>Они вообще всюду ?
Ну не совсем, внутри транзисторов думаю их нет, например.
> Вот на бумажных обоях, на лакированном столе они есть ?
Да.
> Или нужно обязательно чтобы от меня отлетел кусочек органики, которым они и будут питаться, а когда съедят то все передохнут ?
От тебя эти кусочки и так летят постоянно как листва с дерева. Эти кусочки - и есть пыль. А бактериям много не надо, + органика есть и не только с твоего тела. А некоторые вообще умудряются жрать, например, железо(или ржавчину, короче извлекать энергию из окисления металлов).

>Если есть бактерии, которые едят не оганические соединения, то почему они не съедают вообще всё вокруг ?
Потому что не все ля них съедобно и питательно.

>Я прокалил железку в стерильных условихя, будут ли на ней бактерии, откуда возьмутся ?
Изначально нет, возьмутся из воздуха, с твоих рук(которыми ты касаешься железки), от других предметов.

Аноним 09/02/18 Птн 15:31:18  430721
Что будет, если опреснеет мировой океан? Не частично, как в случае с полярными шапками, а совсем. Полностью. Представьте, что условный маг кастанул условное заклятие и стала пресной вода.
Аноним 09/02/18 Птн 18:17:55  430726
>>430721
Ничего страшного, биосфера вымрет только. Включая человечество, конечно.
Аноним 09/02/18 Птн 18:43:09  430728
>>430721
Если быстро - рыба может передохнуть, не все виды пресноводные.
А если медленно (снижение концентрации солей) - то наверняка эволюционируют, накапливая в клетках достаточно минералов.
Но если ты вообще имеешь в виду дистиллированную воду, то жить в ней морским обетателям не получилось бы, она бы вымыла все соли,
а нервы работают за счёт ионов минеральных солей - ионы натрия, ионы калия, и прочее всякое: https://ru.wikipedia.org/wiki/Потенциал_действия
Аноним 09/02/18 Птн 19:46:32  430733
Могут ли животные/растения одного вида, но разных сортов/пород генетически отличаться больше, чем разные виды?
Аноним 09/02/18 Птн 20:12:56  430736
>>430733
токсономное исследование??
Аноним 09/02/18 Птн 20:54:36  430740
>>430728
Ок, спасибо.
>>430726
И так это знал. Хотел уточнить детали.
Аноним 10/02/18 Суб 01:52:35  430784
>>429247 (OP)
Реквестирую книг, типа этих: https://news.rambler.ru/business/38936647-ot-ibm-do-uber-istoriya-krupneyshih-kompaniy-v-33-knigah/
где в кратце описываются фазовые переходы, ведущие к развитию научно-технического прогресса,
или каких-либо прорывных инноваций, или открытие целых областей науки, а также механизмов образования смежных наук.
Аноним 10/02/18 Суб 01:59:19  430785
>>430733
Что значит генетически отличаться больше или меньше.
А вообще, теоретически, если взять днк, перехерачить все гены, которые не экспрессируются, а потом вырастить из этого организм, то по идее должно получиться практически то же самое, что и было.

А, еще прмер придумал. У тебя представителями вида один хрен считаются все живущие в данный момент. Соответственно если взять популяции из миллиардов организмов и из сотни, то в первой разнообразие будет гораздо больше. Особенно если во второй специально поубивать часть.
Аноним 10/02/18 Суб 12:15:01  430809
Почему переменный ток может идти по одному проводу (с фазы розетки на землю), а постоянный нет(с плюса или минуса батарейки на землю)?
Аноним 10/02/18 Суб 12:39:16  430813
>>430657
Библия.
Аноним 10/02/18 Суб 17:59:04  430834
Насколько научна психология, логопедия?

С одной стороны мне с детства говорили что "психолог - это такой же врач как и все". С другой стороны это всегда был врач, который никаким образом мне не помог. Из всех, к которым я ходил. ЛОР вылечил мне гайморит, неврологи смогли поправить неврит лицевого нерва. А вот психологи с логопедами так и не смогли особо поправить мою дислексию.
Да, возможно это не лечится вообще. Но блин, они заёбывали меня много лет.

А частные психологи ещё и деньги за это брали.

По мне, так это кучка шарлатанов. Помимо личного опыта прохождения этих врачей есть ещё пара вещей, которые меня склоняют так считать.

1) Психология пестрит беллетристикой
2) Почти все психологи, которых я видел, были склонны верить в нумерологию, астрологию и прочую ересь.
3) Психологию преподают не в медицинских вузах.
4) Программа очень сильно зависит от государственного строя. В СССР преподавали скромность, взаимовыручку и жертвенность. В США преподают успех, выгоду, инициативность, самоуверенность.
До сих пор помню что СССРовских учебниках было написано что самооценка бывает завышенная, заниженная и нормальная. И завышенная ощущалась хуже, чем заниженная. Сейчас, похоже, что чем выше - тем лучше.


Так вот, есть ли какие-то пруфы или аргументы в одну или другую сторону?

PS: Естественно "преподают в универах" для и "признанно научным сообществом" для меня аргументом не являются. Теология тоже преподаётся в универах. А на журфаках в некоторых вузах есть богословие.
Аноним 10/02/18 Суб 19:08:25  430838
>>430834
>психологи с логопедами так и не смогли особо поправить мою дислексию
Ты бы ствоей проблемой к программистам пошел или шахтерам. Психология никаким боком к дислексии не стоит, а логопеды так вообще правят дефекты речи.

>>430834
>4) В СССР преподавали скромность, взаимовыручку и жертвенность. В США преподают успех, выгоду, инициативность, самоуверенность.
Ты вообще хнаешь что такое психология? Так блджад и преподают на парах по психологии жертвенность, да?
>"признанно научным сообществом" для меня аргументом не являются
Поиграть в убеждалки пришел? Тогда иди прямо нахуй из этого раздела.

>Теология тоже преподаётся в универах
Потому что теология - наука о религиях, я бы эту хуйню частью обязательной программы сделал, если такая есть. Учитывая как эта хуйня маячит на протяжении всей истории человечества, знать и изучать этот феномен - прямая обязанность моченых.
>журфаках в некоторых вузах есть богословие
В реалиях современной России - это вполне такой полезный скил для любой журнашлюшки.

Ты или дебил, или зеленый. Покормил, в общем.
Аноним 10/02/18 Суб 20:49:40  430850
Салют, ананасы. Думаю в рамках благотворительности прикупить научпопа для местной библиотеки, что посоветуете?
Аноним 10/02/18 Суб 21:10:11  430855
>>430809
>а постоянный нет
как проверял?
Аноним 10/02/18 Суб 21:14:39  430856
>>430850
биология - Савельев, физика - Ацюковский, история - Фоменко
Аноним 10/02/18 Суб 22:08:34  430862
>>430856
Ну нитраль, я серьёзно.
Аноним 10/02/18 Суб 23:28:42  430870
Посоны: я вот учу физику. Решаю там задачки, теорию почитываю. Возник вопрос. Что важнее, нарешать как можно больше задач по теме или решил пару задач и анализируешь ответы до посинения (до озарения). Просто сколько бы я ни решал кинематику, я не могу сходу ответить на многие вроде бы простые вопросы. Ну например, будет ли скорость сближения 2 автомобилей постоянной, если у них одинаковая по модулю ускорение, но один разгоняется, а другой тормозит. Я могу конечно посчитать, но вроде ответ на такой простенький вопрос должен быть очевидным, как камень надает на землю, или грузик на нити отклоняется сильнее, если быстрее крутить нить. В общем, как усилить понимание, чтобы многие вещи стали самоочевидны?
Аноним 11/02/18 Вск 00:22:59  430872
>>430855
Ну в случае с переменным током полно пруфов использования такой однопроводной передачи через землю (например, индикаторная отвертка). А вот пруфов с батарейкой и землей я что-то не встречал. Поэтому и делаю вывод, что тут такая схема работает.
Аноним 11/02/18 Вск 00:44:19  430874
Как работает переменный ток?
Тип, понятно, что постоянный ток - это упорядоченное движение электронов, от минуса к плюсу
А как движутся заряженные частицы в случае переменного тока?
Аноним 11/02/18 Вск 00:49:16  430875
>>430495
Плазма - это ионизированный газ
Точно так же, как бывают разные газы, бывают разные плазмы
Собственно, они и светятся разными цветами
Аноним 11/02/18 Вск 05:22:32  430887
Какие инструкции есть у космонавтов , если какой-то член экипажа сойдет с ума?
Аноним 11/02/18 Вск 05:23:19  430888
>>430874
Да ты и про постоянный ток почти не знаешь.
Аноним 11/02/18 Вск 12:56:12  430905
Ниже определенного уровня есть слой грунта с постоянной плюсовой температурой. Если сверху _не_такая_же_температура_, теоретически возможно построить на базе разности температур тепловой двигатель (а значит делать дармовое электричество). Ясно, что КПД будет очень низким (но это не должно волновать: всё равно халява). Почему нет таких систем ещё? Каким-нибудь отшельникам типа Лыковых они бы сильно пригодились.
Инбифо: низкая удельная мощность ("Ватт на квадратный метр" грунта, под которым проложена труба с теплоносителем). Есть ли теоретические оценки мощности такого двигателя?
Аноним 11/02/18 Вск 13:37:25  430906
>>430905
Это называется тепловой насос. Их используют, для обогрева частных домов. Проблемма в том, что им надо многго места, лучше иметь сразу несколько на участке чтобы земля не промерзала, зависит от грунта еще вроде
Аноним 11/02/18 Вск 13:38:57  430907
Кто тако Мунин?
Аноним 11/02/18 Вск 13:45:42  430908
>>430907
Ворона такая.
Аноним 11/02/18 Вск 14:47:27  430909
>>429247 (OP)
Двач это революция?
Изменение своего генокода, в прямом эфире,
недорого без смс.
http://stm.sciencemag.org/content/10/426/eaan4488 сам это не читал, сложна Возможность разик уколовшись излечится от 95% видов рака. Не сейчас, но типа скоро.
https://gmpnews.ru/2017/11/gennaya-terapiya-ot-spark-therapeutics-mozhet-byt-odobrena-v-nachale-2018-goda/ уже прям сейчас заплатив всего 1 000 000 бакинских, некоторые слепые могут стать зрячими изменив свой генокод.
Вроде еще лечение диабета в процессе по той же процедуре.

А теперь вопрос.
Почему нет хайпа в новостях?
Я все проспал и это не новость? Это невозможно?
Генокод это мы, изменить можно практически что угодно, какие перспективы! Но все молчат. Странно.
Вчера один анон создал тред в /b , и там был этот вопрос, ответ я не нашел. Дискасс.
Аноним 11/02/18 Вск 15:34:23  430910
>>430906
> Это называется тепловой насос.
Тепловой насос - это "двигатель наоборот", так что это не совсем то.
Аноним 11/02/18 Вск 17:46:01  430919
>>430838
>Ты бы ствоей проблемой к программистам пошел или шахтерам. Психология никаким боком к дислексии не стоит, а логопеды так вообще правят дефекты речи.

У меня были проблемы(И да, я в 9-10 лет не знал чем они вызваны и что это называется дислексия. Даже не имел представления) и меня отправили к школьному психологу. И любой нормальный врач в таком случае бы выписал направление к другому врачу. Логопеды правят дефекты речи, правда. Но я все звуки произношу правильно, не заикаюсь и не картавлю. Поэтому направили к психологу. К логопедам я тоже ходил, и не слишком помогло.

> Так блджад и преподают на парах по психологии жертвенность, да?

Да, ёбаный в рот, преподают.
Так же как и любую гуманитарную науку.
Литература зависит от режима(читать ли Солженицына, или считать пидором)?
Историю преподают в зависимости от режима.
Даже ёбаное право зависит от режима.

>Потому что теология - наука о религиях
Да-да, попизди мне тут
Аноним 11/02/18 Вск 18:09:57  430922
>>430909
>Возможность разик уколовшись излечится от 95% видов рака. Не сейчас, но типа скоро.
Бред.
>уже прям сейчас заплатив всего 1 000 000 бакинских, некоторые слепые могут стать зрячими изменив свой генокод.
При редкой мутации, которая дает врожденную слепоту. Таких в Америке, насколько помню, человек 200. И не все могут позволить.

>Почему нет хайпа в новостях?
Вот когда допилят, тогда и приходите.

Аноним 11/02/18 Вск 18:20:10  430923
>>430887
ПОЧЕМУ ВЫ НЕ ОТВЕЧАЕТЕ НА МОЙ ОТВЕТ?!
Аноним 11/02/18 Вск 19:00:49  430930
>>430922
>Вот когда допилят, тогда и приходите.
А разве это не начало?
>Возможность разик уколовшись излечится от 95% видов рака. Не сейчас, но типа скоро.
>Бред.
А ты прочитал ссылку?
>При редкой мутации, которая дает врожденную слепоту. Таких в Америке, насколько помню, человек 200. И не все могут позволить.
Ну это ведь неважно, ну. Главное перспективы.
Хочу суперсилу, энергичность 18 часов в день и большее количестов эндорфинов вырабатываемых по команде.
Аноним 11/02/18 Вск 21:54:48  430951
>>430930
>и большее количестов эндорфинов вырабатываемых по команде.
Это не перспективы, сынок. Это "я хочу стать наркоманом и торчать бесконечно и бесплатно".
Аноним 11/02/18 Вск 21:57:06  430952
>>430923
Потому что соси хуй, быдло вот почему.
Аноним 11/02/18 Вск 21:57:50  430954
>>430919
>Да, ёбаный в рот, преподают.
>Так же как и любую гуманитарную науку.
>Литература зависит от режима(читать ли Солженицына, или считать пидором)?
>Историю преподают в зависимости от режима.
>Даже ёбаное право зависит от режима.
А сейчас ты идёшь, открываешь любой совковый учебник по математике, физике, химии и читаешь там в предисловии восхваление политики партии на 20 страниц, а во вступлении перед каждым разделом читаешь благодарности членам политбюро, ёбаный ты пёс.
Аноним 11/02/18 Вск 22:03:02  430956
>>430954
>там в предисловии восхваление политики партии на 20 страниц
Не 20, а пяток максимум. Эх, жаль, сейчас такого нет. Там всегда были мотивирующие строчки вроде "наука шагает в светлое будущее коммунизма", "народ делает успехи тут и там", "ударными темпами к победе", "качество образования растет". Сейчас откроешь учебник, а там унылое предисловие, где автор хвалит себя, благодарит жену, желает успехов детишкам. Тьфу, блядь. Где сокрушительная мощь идеологии?
Аноним 12/02/18 Пнд 04:29:26  430997
>>430951
для меня это перспективы, потому что моя нынешняя депра меня не устраивает.
Аноним 12/02/18 Пнд 13:39:32  431012

>>430919
>Да, ёбаный в рот, преподают.
>Так же как и любую гуманитарную науку.
Я ваша новыя учительница русского языка. Сегодня будем учить глаголы. Существуют глаголы "жертвовать" и "пожертвовать". Других глаголов в русском языке нет.
Ебать ты дебил.

>Да-да, попизди мне тут
Не смог в определения, ясно. Свободен, школьник.
Аноним 12/02/18 Пнд 16:00:20  431021
>>430809
Честно, я сам в этой теме не силён, но для себя нашёл понимание такое: Земля это ёмкость. Переменное напряжение меняет свой знак и поэтому ток идёт половину периода в землю, а вторую половину обратно. Батарейка же источник постоянного напряжения, когда ты к её полюсу прислоняешь проводник, он сам становится таким же полюсом, т.е. заряды перераспределяются, это тоже ток, но перераспределение происходит очень быстро и наступает баланс, при котором нет направленности двжиения у электронов, поэтомуток уже не регистрируется.(ток это именно нпаравленное движение, да ?). Чем больше Вольтаж батарейки, тм больше будет размер куска земли который станет её полюсом,а дальше просто не пробъёт сопротивление земли.
Аноним 12/02/18 Пнд 17:24:38  431027
Сап саентач. Скажи, если я вдруг окажусь в произвольной точке солнечной системы, отличной от точки, вблизи которой находится земля (т. е. вне пределов места, где земля создает гравитационное ускорение более 1/10000 g, то есть в сотне радиусов земли), смогу ли я принять сигналы от межпланетных космических аппаратов?
Аноним 12/02/18 Пнд 17:31:44  431028
>>430856
>>430850
Что на полку "теология" поставим?
/b/иблиотека, лол.
Аноним 12/02/18 Пнд 17:37:08  431029
>>430888
О токе вообще мало кто знает, абстрактная проекция, модель-хуедель.
Аноним 12/02/18 Пнд 17:49:41  431031
>>430874
>это упорядоченное движение электронов, от минуса к плюсу
Представь забитый людьми вагон метро. Это типа электроны. Если из одной двери зашли двое вышли трое, а из другой зашли двое и вышли двое. То это уже ток.
Они у тебя там "хороводы" не кружат. А если вспонимнить про уровни и электронную плотность, всё становится максимум запутанно
Аноним 12/02/18 Пнд 20:34:43  431043
Что такое холст зелёного цвета? Значит, что из всех лучей спектра, падающих на такой холст, отражаются лучи, соответствующие зелёному цвету, остальные поглощаются. Так?

Но, если отражаются синий и жёлтый в равной степени, тоже получается зелёный. Не обязательно требовать отражения чистого зелёного цвета. Он может и не отражаться вообще.

Помогите понять этот момент, пожалуйста.
Аноним 12/02/18 Пнд 20:54:29  431047
>>430809
Ты неправ.
Аноним 12/02/18 Пнд 21:08:20  431049
>>431043
Что понять? Что глаза не идеальный прибор, который видит суть вещей как они есть?
Аноним 12/02/18 Пнд 21:15:07  431050
>>431049
Понять отражает ли он зеленый или синий и желтый, превращающийся в зеленый.
Аноним 12/02/18 Пнд 21:16:02  431051
>>431050
Возьми спектрометр.
Аноним 12/02/18 Пнд 22:46:07  431055
blob (20Кб, 608x224)
>>429247 (OP)
Анон, является ли ионообменные мембраны и анионные смолы - электрическим демоном Максвелла в проводниках второго рода?
Поясню: К проводникам второго рода относят проводники с ионной проводимостью (электролиты).

Если смотреть на пикрелейтед, то красные молекулы - горячие, синие - холодные.
И тепло передаётся от горячих тел - к холодным.
Демон Максвелла же - делает наоборот, он горячие молекулы пропускает только к горячим, а холодные - к холодным.
Разделение горячих молекул и холодных во смеси их - снижает их энтропию.

А теперь, рассмотрим ионный раствор...
Есть анионы, и есть катионы. Они в электролите смешаны. Энтропия максимальна в растворе.
Ток протекает от положительного полюса к отрицательному (т. е. имеет направление, также как и теплообмен - от горячих тел к холодным). А ионообменная мембрана разделяет ионы в растворе, снижая их концентрацию по разные стороны мембраны.
Таким образом в растворе образуется разность зарядов, обусловленная различной концентрацией ионов.

Казалось бы, ионообменная мембрана не потребляет энергию, она просто либо пропускает, либо не пропускает ионы,
но скорость раздеения зарядов мембраной в растворе - зависит от температуры раствора.
Это значит, что рост тепловой энергии раствора ускоряет разделение зарядов - и получается термоэлектрогенератор.
Ок. А что если не нагревать раствор, а просто оставить его при обычной температуре на длительное время.
Разделение зарядов происходит, но за счёт какой энергии? Что порождает ЭДС источника?
Является ли система из мембраны чем-то наподобие псевдовечного двигателя - двигателя 2-го рода?
И да, что если собрать всё это в одну пиздатую батарею и попеременно разряжать одни заряженные мембраной ионисторы,
а другим дать возможность зарядится за счёт разделения ионов мембраной - получается самовосстанавливающаяся батарея,
с динамически-изменяющимися источниками энергии (отдельные резервуары с ионными растворами и ионообменными мембранами),
но сама батарея наверняка должна была бы выдавать постоянную и стабильную мощность, несмотря на её громоздкость.
Интересно, откуда бы бралась там эта мощность и энергия?

И да, вот ещё... Так просто ток не снять из разницы зарядов в ионном растворе, или всё-же можно?
Т. е. ток в проводнике второго рода - надо ещё конвертировать в ток в проводнике первого рода (электронный) - ну как в ионисторах.
Короче, тут сереневый шарик, а тут синий.

Аноним 13/02/18 Втр 01:05:27  431057
>>431050
проверь светофильтром
Аноним 13/02/18 Втр 05:07:29  431067
>>429247 (OP)
Что будет если в эрбий долбануть дейтроном?
Аноним 13/02/18 Втр 08:57:19  431081
Если микроволны прогревают жратву на глубину см-другой, есть ли диапазон частот, прогревающий и толстенные шматы? Чтоб например сунул в радар бычью ногу, она вся за пять минут до 60 прогрелась, ещё пару минут наводишь сверху гриль - и готово.
Аноним 13/02/18 Втр 12:03:16  431084
ink.png (58Кб, 2048x1098)
Хочу вывести формулу: вероятность перекрытия колоний, выросших на чашке со средой. А потом вероятность, что с помощью робота я отберу колонии, не касаясь друг друга.
Аноним 13/02/18 Втр 12:58:32  431085
У кого-нибудь есть слайды лекций Дубынина? Смотрю сейчас общий курс лекций осени 2015, но в принципе пригодились бы слайды всех курсов, т.к. другие, скорее всего, тоже смотреть буду.
Аноним 13/02/18 Втр 13:07:01  431086
>>431081
Наверное интенсивность падает, с глубиной проникновения, и если тупо увеличить мощность, наружные слои загорятся, а центр один хуй останется холодным.
Аноним 13/02/18 Втр 15:26:41  431090
анончики объсните как можно доступнее как вычеты считать, очень нужно
Аноним 13/02/18 Втр 16:03:49  431095
>>430850
Первое, что в голову пришло: Дробышевский два тома "Достающее звено", Марков два тома "Эволюция человека" + "Рождение сложности", Рамачандран "Мозг рассказывает", Несса Кэри "Эпигенетика", Сапольски "Психология стресса", Ридли "Геном".
Аноним 13/02/18 Втр 18:00:21  431099
>>431055
>А ионообменная мембрана разделяет ионы в растворе, снижая их концентрацию по разные стороны мембраны.
>Таким образом в растворе образуется разность зарядов, обусловленная различной концентрацией ионов.
Ты куку? Ионообменная просто заменяет одни ионы другими. Термодинамику кстати не нарушает. (ни одна химическая реакция её не нарушает.)
Аноним 13/02/18 Втр 18:36:52  431101
>>431095
Спасибо. Я думаю ещё Никитина "Происхождение жизни" купить, заодно и сам прочитаю.
Аноним 13/02/18 Втр 20:01:03  431107
Что за горизонтом событий?
Ёбаная физика, в рот ебал, это пиздец какой-то
Аноним 13/02/18 Втр 20:03:00  431108
>>431107
Слетай да узнай. Потом расскажешь.
Аноним 13/02/18 Втр 20:18:01  431109
3xl0m3aw8Mw.jpg (18Кб, 405x399)
>>431108
Но я не могу, у меня лапки
Аноним 13/02/18 Втр 21:55:51  431110
>>431109
Тогда вот тебе ответ: все что угодно.
Аноним 13/02/18 Втр 22:47:27  431111
Есть больные раком П.Ч., иногда приходится его ампутировать. У меня вот вопрос: у них же желание ебацца есть, а возможности его удовлетворить - нет ну тот же нофапафон, только вечный. И чем всё кончается? Мозг адаптируется и желание пропадает, или им приходится ещё и яйца отрезать?
Аноним 13/02/18 Втр 23:20:41  431112
UnfinishedUnfin[...].gif (1044Кб, 254x213)
>>431111
Мозг адаптируется и ты сможешь кончать от другой стимуляции.

Безотносительно этого, в медицине случаи, когда с мужчиной что-то случилось твёрдо и чётко от полового воздержания, неизвестны.
Аноним 14/02/18 Срд 07:06:00  431131
>>431084
У меня от тебя аксиома выбора
Аноним 14/02/18 Срд 07:09:57  431133
>>431090
Хули там считать? Берешь свою зарплату за год, считаешь 13% от нее - максимальный лимит вычета. Так же с остальными доходами.
Аноним 14/02/18 Срд 10:49:49  431154
>>431131
Смотри что получилось.
Сделал вот по этому гайду https://stats.stackexchange.com/questions/17954/probability-of-circles-intersecting свою симуляцию. Можно ее оптимизировать?

x <- runif (10000, 0, 3.141617.5)
y <- runif (10000, 0, 3.141617.5)
r <- runif (10000, 0.35, 0.75)
C <- cbind(x,y,r)

sol2 <- function(i) {
vec<-c()
for (j in 1:10000) {
vec[j]<-(C[i,1]-C[j,1])^2+(C[i,2]-C[j,2])^2 <= (C[i,3]+C[j,3])^2
}
((10000-sum(vec, na.rm=TRUE))/10000)^24)
}

sol4<-c()
for (i in sample(c(0:10000), 9999, replace = TRUE)){
sol4 <- append (sol4, sol2(i))
}
mean(sol4)
Аноним 14/02/18 Срд 11:57:18  431169
fem.png (41Кб, 794x632)
MpT2rXtA3mU.jpg (203Кб, 860x1080)
Здравствуйте. Верно ли утверждение, что женщины не производят вклада в эволюцию? Самые тупые и слабые самцы постоянно отсеиваются, а женщина оставит потомство при удовлетворении одного лишь фактора: наличие пизды.
Если вышесказанное верно, то почему общество не признает самок суррогатом человечества, с соответствующим отношением к последним? Почему есть неадекватный феминизм, с алогичными утверждениями и требованиями, но отсутствует справедливый и очевидный маскулизм?
"Не дают, вот и бесишься" - обычный ответ, на подобный вопрошения, "ну кто с убогим станет спорить, что он убогий" - отмахиваются другие. Но уже сейчас неадекватные самки, с избытком свободного времени, влияют на мировую культуру, законотворчество, искусство, укореняя лицемерие, ложь, несправедливость, чем оказывают вредное, тормозящее прогресс влияние.
Почему пропагандируется культ женщины? А быть может они уже и правят миром? Ведь каждой девушке известно, как легко можно манипулировать практически любым мужчиной, используя его примитивнейшие инстинкты.
Аноним 14/02/18 Срд 11:59:39  431170
>>431169
Чет кекнул с второй картинки.
и заплакал, два золота межнара
Аноним 14/02/18 Срд 12:09:54  431172
>>431169
> пизды
> вышесказанное верно
Ты со стилистикой определись, калека.
Аноним 14/02/18 Срд 12:22:09  431177
>>431169
>Верно ли утверждение, что женщины не производят вклада в эволюцию?
Не верно.

Аноним 14/02/18 Срд 12:40:32  431180
>>431172
Толстота - традиция двача, пишу, как хочу, не нравится - не отвечай.

>>431177
Естественно вклад есть, как вклад обуви в результат спортсмена. Имеется ввиду, что он несоизмеримо мал, и незначителен, в сравнении с мужским.
Аноним 14/02/18 Срд 12:43:58  431181
>>431180
>Имеется ввиду, что он несоизмеримо мал, и незначителен, в сравнении с мужским.
Доказывай.
Аноним 14/02/18 Срд 13:21:47  431185
>>431181
Самцы, большинства видов млекопитающих, конкурируют между собой, ради права спариться и оставить потомство. Глупые и слабые самцы не могут оставить потомство, за исключениями конечно, но преимущественно наиболее плодовитые, это самые умные, сильные самцы, те, чей организм наиболее приспособлен к существованию.
Хуевое здоровье? Тупой? Нерешительный? Отправляешься на помойку эволюционной истории.
Самке же, для того, что бы вложитсья в генофонд необходимо одно - дожить до пубертата. Тупая, больная? Не страшно! Недостатки самки будет разбирать уже её потомство, при чем преимущественно самцы. Естественно птомство больной самки будет дохнуть чаще, но оно будет.

Признаю, что был неправ выше, самки не то, что вносят недостаточный вклад в эволюцию, они её тормозят и затрудняют, так как возможность передать плохие гены потомкам у них гораздо выше.
Аноним 14/02/18 Срд 13:36:48  431186
>>431185
Для эволюции не важно есть или нет потомство.
Только качество.
Потомство плохих самок - плохое - оно все равно бесполезно для развития, не дав ничего виду, как не даст потомство плохих самцов.
Но ты не в том ключе рассматриваешь роль самок в эволюции, проецируя остальных млекопитающих на хомосапиенсов.
Присутствие самок в популяции заставляет самцов КОНКУРИРОВАТЬ. Дальше додумай сам, мне лень тебе разжевывать.
Аноним 14/02/18 Срд 14:05:20  431188
>>431186
>Для эволюции не важно есть или нет потомство.
Собственно и это уже достаточно убедительно, но есть замечание:
>Присутствие самок в популяции заставляет самцов КОНКУРИРОВАТЬ
Присутствие еды, а точнее её необходимость, тоже вынуждает конкурировать. В принципе самок можно выкинуть вообще, однополые самовоспроизводящиеся организмы будут отлично конкурировать за еду, и эволюционировать. Но ведь они все перевымерли на заре зарождения жизни, возразишь ты. Да, в те, ебучие времена выгоднее была комбинация генов, но и организмы были в более равных условиях, никаких ебучих конкурсов и гонок.
Аноним 14/02/18 Срд 14:25:24  431194
>>431188
>В принципе самок можно выкинуть вообще, однополые самовоспроизводящиеся организмы будут отлично конкурировать за еду, и эволюционировать.
Это всего лишь твои маняфантазии против истории эволюции биосферы на планете.
Смысл в том, что те, кто побеждают в конкуренции - доминируют.
Так вот как показала практика - единственный критерий истины - пока что вырвались вперед в гонке двуполые.
Аноним 14/02/18 Срд 14:54:35  431198
smugfrizzy.gif (1801Кб, 420x236)
>>431194
>пока что вырвались вперед в гонке двуполые.
>95% биомассы на планете бактерии и водоросли
Ну ок, вырвались так вырвались, что уж тут поделаешь. Всех победили, все ниши заняли, такие молодцы.
Аноним 14/02/18 Срд 15:06:02  431201
>>431198
Для доминирования нет необходимости занимать все ниши. Нужно занять верх пирамиды.
Аноним 14/02/18 Срд 15:25:13  431204
>>431201
На верху пирамиды ВИЧ, столбняк, энцефалит. Тоже достаточно бесполые сущности.
Аноним 14/02/18 Срд 15:40:12  431206
>>431204
>На верху
Когда из-за них хотя бы прекратится рост населения основного доминирующего вида, тогда и приходите.
А пока они сидят под шконкой, потому что потребление ими энергии в масштабах планеты КРАЙНЕ МАЛО.
Аноним 14/02/18 Срд 15:55:13  431209
>>431206
>потребление ими энергии
Разумеется, в потреблении энергии тоже с огромным отрывом выигрывают океанические водоросли.
Какой ещё новый манякритерий ты выдумаешь, чтобы не соснуть? Я заинтригован.
Аноним 14/02/18 Срд 16:19:22  431214
>>431209
Угу и растения с их 0,1% КПД.
Удачи им в доминировании.
Аноним 14/02/18 Срд 16:22:38  431217
>>431194
Какие фантазии они и сейчас живут и эволюционируют. И вообще я говорил про вклад самок, в эволюцию млекопитающих.
Аноним 14/02/18 Срд 16:26:06  431219
>>431198
Окей, двуполые доминируют среди многоклеточных животных. Доволен?
Аноним 14/02/18 Срд 16:27:29  431220
>>431214
КПД цивилизации вообще отрицательный. Половину ресурсов проебём в космос, вторую половину в сложные химически инертные соединения на свалке. А когда кончатся углеводороды, вымрем до своего естественного уровня в несколько сотен тысяч. Оце так перемога!
Аноним 14/02/18 Срд 16:31:47  431223
>>431220
Лол, че кончится? Где? Представил сейчас неопределенного пола личностей, с разноцветными волосами, с табличками "сейв зе хидросорбонате".
Аноним 14/02/18 Срд 16:34:13  431224
>>431099
Ну, разность потенциалов создаётся же ионообменной мембраной, заряды разделяются в растворе.
Я просто смотрю на неё как на демона Максвелла - потому что она порождает ЭДС в ионном растворе,
и эта ЭДС - является ЭДС обратной притяжению заряженных ионов по разные стороны раствора,
кроме того, она, эта ЭДС - может увеличиваться при повышении температуры раствора.
Ну ионы быстрее двигаются при повышении температуры же,
а значит быстрее на этой мембране и разделяться должны.
К тому же без мембраны ионы смешались бы, а мембрана наоборот их разделяет ещё больше.
Ионы стремятся притянутся, но не могут, им мешает мембрана. Она ещё больше заряжает разные стороны раствора.

То же самое примерно и при теплообмене происходит.
Горячие молекулы двигаются к холодным - естественным образом, также как и положительные ионы к отрицательным,
а холодные молекулы - проникают горячим, разгоняясь сами и замедляя их - также как и отрицательные ионы проникают к положительным.
Но демон Максвелла делает наоборот, он концентрирует холодные частицы на холодной стороне, а горячие - на горячей.
Когда коробка без демона Максвелла - энтропия максимальна, это то же самое, что если мембраны нет и смесь ионов однородна,
но когда демон Максвелла работает - разные части коробки по-разному нагреты, также как и ионы разного знака при наличии мембраны разделены.
Энтропия раствора с мембраной при этом должна быть минимальна, также как и с демоном Максвелла.

Но прикол в том, что при этом демон Максвелла по определению не использует энергию для организации энтропии, оттого и нарушает второй закон термодинамики.
>Энтропия для системы, состоящей из правой и левой части сосуда, в начальном состоянии больше, чем в конечном,
>что противоречит термодинамическому принципу неубывания энтропии в замкнутых системах (см. Второе начало термодинамики).
Но
>Парадокс разрешается, если рассмотреть замкнутую систему, включающую в себя демона Максвелла и сосуд.
>Для функционирования демона Максвелла необходима передача ему энергии от стороннего источника.
>За счёт этой энергии и производится разделение горячих и холодных молекул в сосуде, то есть переход в состояние с меньшей энтропией.
В этом случае второе начало термодинамики не нарушается, и работает негэнтропия.
В самом общем смысле этот термин противоположен по смыслу энтропии и означает меру упорядоченности
и организованности системы или качество имеющейся в системе энергии.

Поэтому я и говорю, что ионообменная мембрана чем-то похожа на демона Максвелла, особенно если нагреть раствор (подвод энергии).
Ведь за счёт увеличения внутренней энергии должна увеличится и скорость разделения ионов в растворе,
а значит упасть энтропия (мера беспорядка ионов в растворе).
Если конечно смотреть на энтропию - как на меру беспорядка:
https://ru.wikipedia.org/wiki/Энтропия_в_классической_термодинамике#.D0.9F.D0.BE.D0.BD.D0.B8.D0.BC.D0.B0.D0.BD.D0.B8.D0.B5_.D1.8D.D0.BD.D1.82.D1.80.D0.BE.D0.BF.D0.B8.D0.B8_.D0.BA.D0.B0.D0.BA_.D0.BC.D0.B5.D1.80.D1.8B_.D0.B1.D0.B5.D1.81.D0.BF.D0.BE.D1.80.D1.8F.D0.B4.D0.BA.D0.B0
Аноним 14/02/18 Срд 16:41:32  431225
почему в холодный день виден пар при выдыхании?
Аноним 14/02/18 Срд 16:55:13  431229
>>431225
Он конденсируется в туман.
Аноним 14/02/18 Срд 17:34:32  431235
>>431223
В перспективе. Это конечный ресурс, на нём держится 99% логистики и экономики.
Аноним 14/02/18 Срд 17:37:26  431237
>>431154
x <- runif (10000, 0, 3.141617.5)
y <- runif (10000, 0, 3.141617.5)
r <- runif (10000, 0.35, 0.75)
C <- cbind(x,y,r)

func.c <- function(i,j) {
(C[i,1]-C[j,1])^2+(C[i,2]-C[j,2])^2 <= (C[i,3]+C[j,3])^2
}
sol.c<-as.matrix(apply(C, 1, func.c))

sol.c1 <- matrix(unlist(sol.c), ncol = 10000, byrow = TRUE)


sol.c2<-c()
sol.c2 <- rowSums(sol.c1)
sol.c3 <- ((10000-sol.c2)/10000)^24

ОПТИМИЗИРОВАЛ

вот теперь не понимаю последней проблемы https://stats.stackexchange.com/questions/17954/probability-of-circles-intersecting :
1. вычисляем, перекрываются ли любые 2 рандомных круга из 10000 - примерно в 1% перекрываются
2. так как кругов 25, то возводим в степень n-1
3. почему теперь я опять должен умножить на количество кругов n, как указано в последнем абзаце?
Аноним 14/02/18 Срд 18:19:31  431243
>>431224
Я не буду тебе отвечать.
1)Ты пытаешься учить меня термодинамике. Зачем?
2)Ты пишешь о разности зарядов в растворе. Емае. Ну ты хоть реакции напиши. Может заметишь, что электронейтральность сохраняется. посмотри на тот же электролиз NaCl с мембраной. У тебя на один акт на катоде, один акт на аноде плюс один катион натрия через мебрану проплывает от анода к катоду.
Аноним 14/02/18 Срд 22:06:47  431271
1518635190558.jpg (63Кб, 300x239)
Поясните по-хардкору за разницу между лингвистикой и филологией.
Аноним 14/02/18 Срд 23:29:33  431281
image.png (930Кб, 721x1280)
Схуяли на втором графике две проекции ускорения за один промежуток времени??
Аноним 14/02/18 Срд 23:43:08  431284
>>431185
Охлол. Самка должна потомство выносить, родить (ии неплохо бы ещё в живых при родах остаться), выкормить и защитить. Фэйл на любом из этих этапов - и гены самки дальше не поедут, вот тебе эволюция. Причём речь идёт не только и не столько о "эта самка сдохла во время беременности, всё", сколько о "самки данного типа доводят потомство до фертильного возраста на 1% чаще, чем самки конкурирующего, через 500 поколений конкурирующего не останется".

И потом,именно самка решает, кто из самцов - лох, а кто нет, если самки хуёво решают, то данная популяция будет разбазаривать ресурсы на неоптимальное потомство и отсосёт.
Аноним 15/02/18 Чтв 00:40:03  431293
>>431271
Разница как между обобщением и конкретизацией если в двух словах.
Аноним 15/02/18 Чтв 11:37:30  431323
Поясните за лицевые и затылочные валики. Где они находятся? Желательно показать на черепе. Вот есть надбровные валики, я так понимаю это надбровные дуги. Но про лицевые валики нихуя найти не могу - всё косметическим говном засрано.
Аноним 15/02/18 Чтв 13:12:40  431337
>>431284
Опять же, самка лишь деталь, хоть и важная несомненно, в механизме эволюции, уже лет через 200 необходимость в них отпадет. Суть эволюции - ОТБОР, самки же отбору не подлежат, следовательно влияют на процесс косвенно. По сути - предоставление услуги инкубатора + вброс своих генов. Несомненно, отбор идет и среди самок (сейчас просто ебанутый переход нашей популяции от обыкновенного животного к трансгуманистическому "сверхчеловеку", нынешний человек имеет все признаки первых, но активно фантазирует и стремится ко второму (уже совсем не фантастикой звучит создание эмбриона имея лишь генетический материал/комбинацию генов, минуя яйцеклетки, сперму, и последующее выращивание в искуственном инкубаторе) и совершенно непонятно, как философствовать на эту тему, напишешь "человек" - обоссут, напишешь "животное" - обоссут тоже), как идет и среди бактерий в нашей жопе. Если смотреть биосферу в самом общем понимании, её существование в настоящем виде невозможно, без самых даже незначительных её элементов, и вклад их неоценим.
Если обсуждать эволюцию, отбор - то отбросив все косвенное, останутся лишь носители генов, сумевшие или не сумевшие скопировать самих себя. Самки, в такой системе носят ритуальный, религиозный характер: "хранительницы очага", "прекрасный пол", и тд. Вспоминается изречение некого известного пиздабола: "религия без науки - хрома, наука без религии - слепа", что собственно и верно, в некотором смысле, религия несомненно оказывала влияние на науку, но по факту религия и нахуй не нужна, просто свечку держала когда-то.
Давайте опять обратимся к трансгуманизму: лет через 200, когда засовывание хуя самца в пизду самки, с последующим высиранием из последней лечинки человека, будет даже звучать дикостью, останутся просто люди. Носители генов, из которых рептилоиды (зог/компьютер/сами люди) будут выбирать самые полезные/выгодные для изготовления новых человеков. Особи недопущенные к донорству генов, тоже будут принимать участие в развитии постчеловечества, воспитании личинок, убиранию говна, но без загрязнения генома.
Получается, что мужские особи особо не изменятся, но такого понятия как "женщина" не останется в принципе, никто не станет пилить потомство из непроверенных генов, так как носители их харошие и заботятся.
Естественно вышевысранное обретает смысл лишь для более социальных животных, где играет роль культура и прочая психология. Многие животные действительно вкладывают равный вклад в выживание потомства, но это совсем не применимо к современному человечеству, и тому пиздецу, в который оно стремится. А может вся эта культура - тупик и смерть, как завещал дедушка Ферми.
Аноним 15/02/18 Чтв 14:45:24  431342
Обычная соль увеличивает температуру кипения, а есть ли понижающие её?
Аноним 15/02/18 Чтв 22:22:12  431393
>>431342
>Обычная соль увеличивает температуру кипения, а есть ли понижающие её
Нет.
Следствие из закона Рауля: повышение температуры кипения раствора, раствор закипает при более высокой температуре, так как концентрация молекул растворителя в нём всегда ниже, чем в чистом растворителе, и давление насыщенного пара раствора достигает атмосферного при более высокой температуре.
Аноним 16/02/18 Птн 00:05:53  431397
Мой вопрос связан исключительно с физиологией человека, и ничем больше. Многие кто создавал тульпу сообщают о боли в голове, давлении. И я сам стал это замечать во время форса тульпы, давление появляется в области макушки голове или на правой стороне, еще чувство зуда внутри мозга. С чем это связанно? Хотя бы какие нибудь предположения есть?
Аноним 16/02/18 Птн 00:07:08  431398
>>431323
https://www.biodigital.com/ тут подробнейшая 3D модель человека, может там найдешь что.
Аноним 16/02/18 Птн 00:14:59  431399
>>431169
>вклада в эволюцию?
Эволюция это вообще рандомный процесс, никто никакой вклад не вносит.
>Почему пропагандируется культ женщины?
На мой взгляд, большинство производителей культурного контента мужчины. Большинство мужчин любит женщин, мужчины делают контент для мужчин которые в свою очередь тоже любят женщин. И складывается впечатление что есть какой то культ женщины, и у самих женщин такое впечатление. Оно складывается на основе массовой и бытовой культуре. Но в целом нет никакого культа, сейчас мы на равных стоим, разве что во всяких восточных парашах правит мужчины правят и дрочат на мужественность. И куда это их привело? Никакого культурного прогресса, ничего. Одна война да насилие.
Аноним 16/02/18 Птн 01:02:50  431400
>>431399
>Никакого культурного прогресса, ничего. Одна война да насилие.
Ты ошибаешься, анон. Культурный прогресс нашего вида это и есть война.
Раньше беременным бабам стреляли в живот - теперь им в живот тычут тазером. Раньше чучмеки резали головы просто так - теперь они режут головы на UHD камеры с софитами. Значительный прогресс достигнут.
Аноним 16/02/18 Птн 02:37:24  431403
>>431400
Война конечно дала толчек научному прогрессу. Но сейчас совершенно другая ситуация. А бабахи нечего не создали, лишь пользуются благами запада, но сами продолжают жить в средних веках. Оставь им планету они понастроют миллионы храмов, установят анальный шариат и здохнут вместе с той же планетой причём намного раньше. Их общество вообще не способно к прогрессу, а улучшение только способов убийства своего вида ничего не даст. Как и тот анон со своей мыслью о мужчинах они такие же узколобые и не способные понять и принять факт сложности и многогранность мира.
Аноним 16/02/18 Птн 02:56:08  431405
>>431403
>А бабахи нечего не создали
Ты находишься в заблуждении, если считаешь, что цивилизация это какой-то квест, а условием выполнения квеста является создание х86_64 и фетиша на небритую манду. Бабахи очень сильны и упрямы, и их сила и упрямство это серьёзный залог их выживания.
Аноним 16/02/18 Птн 03:01:06  431407
>>431403
Рассуждаешь как Римляне про германских варваров с севера.

У бабахов был период рассвета, и может еще будет.>>431403
Аноним 16/02/18 Птн 09:44:47  431427
Допустим рядом с домом стоит платформа на пружине. Бросаем с первого этажа на эту платформу тело и пружина сжимается. Потом бросаем с десятого этажа и пружина сжимается сильнее.

Сжатие пружины рассчитывается так:x = F/K то есть зависит от приложенной силы.

НО! По закону ньютона выходит же, что сила в обоих случаях с первым и десятым этажом должна быть одинаковая: F = mg. А значит и пружина должна сжиматься одинаково.

Где подвох?
Аноним 16/02/18 Птн 09:48:01  431428
>>431427
Лол.. ты забыл движение учесть.
Грамотно запиши задачу.
Аноним 16/02/18 Птн 10:08:53  431429
>>431428
Я сказал, что это свободное падение тела, на которое действует только ускорение свободного падения.
Аноним 16/02/18 Птн 10:16:18  431431
>>431427
F = mg - это сила тяжести, действующая на груз (то есть его вес), на столько пружина сожмётся, если груз на неё просто положить, после этого груз будет лежать стабильно на пружине. Но в случае с падающим грузом он не будт лежать стабильно, он отскочит обратно (в простой модели - ровно до того этажа, с которого был скинут). То есть на пружину действуем сила, бОльшая, чем F, причина - наличие у груза собственной скорости, которую нужно погасить. Если хочется посчитать сжатие пружины в такойситуации - это через энергию хорошо делать.
Аноним 16/02/18 Птн 10:17:43  431432
>>431427
Максимальная деформация пружины расчитывается вот так:

х=(mg+sqrt{[mg]^2+mkv^2})/k

Из закона сохранения энергии.
Аноним 16/02/18 Птн 10:20:14  431433
>>431427
>По закону ньютона выходит

А закон Ньютона вообще вот так записывается:

m(d^2x/dt^2)=mg-kx
Аноним 16/02/18 Птн 10:32:33  431435
>>431429
Я говорю сядь запиши дано. Законы Ньютона ебани. не забудь учесть ускорение в обратную сторону при торможении
Аноним 16/02/18 Птн 11:21:26  431438
>>431432
Мне не нужен ответ, я не в школе сижу. Просто хочу понять, как это работает.
Как ты вывел силу? И почему у тебя коэффициент жёсткости пружины находится в числителе под корнем, если в числителе только сила?

>>431431
>>431433
>>431435
Я не настолько могу в теоретическую механику, чтобы это сделать самостоятельно. Пришёл сюда как раз за помощью, чтобы здесь пояснили.
Аноним 16/02/18 Птн 11:52:33  431441
>>431438
Ок. Ты написал F=mg. Но на тело действует еще и сила от пружинки. F=-kx. Беру с обратным знаком, так как они в разные стороны действуют.
Итоговая сила F=mg-kx; За x=0 взял уровнь пустой платформы.
Теперь берем еще один закон Ньютона.
F=ma;
Получаем ma=mg-kx; после момента контакта с платформой. Ок, что получается? Надо теперь отсюда найти x . Когда скорость станет равна нулю. И тут вот уже выходит, что это зависит от скорости с которой тело прилетело. Это очень долгий путь решения, конечно можно проще, воспользовавшись законом сохранения энергии.
Энергия = кинетическая тела+потенциальная тела+потенциальная пружины
E=mv^2/2-mgx+kx^2/2
В момент, когда ты отпускаешь тело. У него только потенциальная энергия от "этажа". Дальше оно падает, энергия переходит в кинетическую.. тело касается платформы и энергия кинетическая переходит в энергию пружины. До полной остановки тела. При этом во всем этом процессе энергия не меняется. Если забить на всякие трения сопротивления.
Далее просто приравниваем эти энергии и выходит простое квадратное уравнение от x.
Аноним 16/02/18 Птн 20:16:38  431499
>>431441
Спасибо тебе, бро. Единственное, что не понятно, это в каких случаях можно применять формулы энергией.

Алсо, есть еще один вопрос по такой школьной или не очень механике.
Допустим, ты ехал на велосипеде и уперся в стенку передним колесом и остановился. После чего встал всем весом на одну педаль (ты весишь 100кг, а значит ты давишь с силой 100 х 9.81 = 981 Ньютон), шатун стоит параллельно земле. Длина шатуна 0.3 метра, а значит момент на оси шатуна 981 х 0.3 = 295 Нм. Я хочу узнать, какая сила действует на каждый зубец звездочки, которая находится на той же оси, что и шатун. Радиус звездочки 0.1 метр, и количество зубцов 30 штук. Никаких потерь не учитываем, и принимаем, что зубцы находятся на том же радиусе, что и у звездочки.
Аноним 16/02/18 Птн 20:41:57  431504
>>431499
>каких случаях можно применять формулы энергией.
Всегда. Энергия и законы Ньютона следствия одного и того же уравнения.
Аноним 16/02/18 Птн 20:44:58  431505
>>431499
Придется всё расписывать.
Аноним 16/02/18 Птн 20:52:39  431507
14288412689010.jpg (34Кб, 567x567)
>>431504
Какого?
Аноним 16/02/18 Птн 20:55:35  431508
>>431507
Принципа наименьшего действия с Лангражевой функцией.
Аноним 16/02/18 Птн 20:59:58  431510
>>431504
Если мы говорим именно о механике - то нет, не всегда, закон сохранения механической энергии имеет ограниченную область применения. Ну то есть если у тебя два пластилиновых шарика столкнулись, слиплись и упали - то закон сохранения механической энергии тут не работает, кинетическая энергия шариков пошла на деформацию шариков и перешла в тепло. Аналогично если трение есть - механическая энергия будет переходить в тепловую. А если тела ещё и заряжены и/или движутся в магнитных полях - то там ещё и энергия электромагнитного излучения может возникнуть.
Аноним 16/02/18 Птн 21:03:01  431512
>>431510
Кто говорил, что я выкидываю потенциальную энергию тел из уравнений энергии?
Ей можно всегда пользоваться. Другое дело, сможешь ли ты записать эти уравнения? И нужно ли вообще это тебе.
Аноним 16/02/18 Птн 21:03:53  431513
>>431512
>внутреннюю энергию
фикс
Аноним 16/02/18 Птн 22:09:07  431521
Arecibomessage.png (27Кб, 320x960)
Какой шанс, что электромангитная волна, содержащая речь Гитлера в 1933 дойдет до принимающей антенны планеты Х в галактике Андромеда? Что может с ней случиться по пути?
Я пытался сам гуглить честно, но простых объяснений нет, только дикий матан.
Аноним 16/02/18 Птн 22:10:54  431523
>>431505
Ну я могу расписать всё, кроме силы на зубцах. Что интересует?
Аноним 16/02/18 Птн 22:15:29  431524
>>431523
Нет ты не понял. Придется сидеть и ебашить систему уравнений со всеми силами на зубцах. Чтобы потом найти.
Аноним 16/02/18 Птн 22:16:39  431525
>>431521
Интенсивность найди. Там наверное единичные фотоны.
Аноним 16/02/18 Птн 22:18:56  431526
>>431524
Зачем система уровнений? Я знаю момент на оси и радиус звезды, а это значит, что я знаю силу на поверхности. Если разделить эту силу на количество зубцов, то это будет то, что я ищу?
Аноним 16/02/18 Птн 22:43:43  431528
>>431526
>Если разделить эту силу на количество зубцов, то это будет то, что я ищу?
боюсь что нет. А если выйдет и так, то это неочевидно
Аноним 17/02/18 Суб 14:42:40  431566
4743989.gif (68Кб, 740x250)
>>429247 (OP)
Если топить урановые ломы в ртути, будет ли золото словно пенка всплывать?
Аноним 17/02/18 Суб 20:21:24  431583
>>431566
https://youtu.be/jkPGW7ilY-s
Аноним 17/02/18 Суб 20:46:37  431584
>>431583
опередил
Аноним 17/02/18 Суб 21:29:48  431587
b91ee5504b0601f[...].jpg (61Кб, 564x645)
>>431583
Про химические превращения рассказали. А ядерные вообще не будут происходить чтоли? извините если использую неправильную терминологию, я гуманитарий, а это тупых вопросов тред
Аноним 18/02/18 Вск 00:30:32  431596
image.png (191Кб, 422x398)
Как будет выглядеть и какими свойствами будет обладать гипотетический астрономический объект, с такой массой, что вторая космическая скорость для него будет меньше скорости света т.е. это ещё не чёрная дыра, но уже больше скорости большинства (99,99%) нейтрино в нём возникающих?
Кароч, что будет с нейтронной звездой настолько массивной, что она не даёт улететь своим нейтрино?
Аноним 18/02/18 Вск 06:15:18  431608
>>431587
Ты про ХУЯС? Нет, его не будет. А ядерные процессы будут происходить только в уране - он делится.
Аноним 18/02/18 Вск 06:17:58  431609
>>431596
Ничего интересного. Разве что солнечного ветра от такой звезды не будет, а радиация будет исключительно гамма-радиацией.
Аноним 18/02/18 Вск 15:27:50  431642
Невозможно передавать информацию быстрее скорости света. Что, если на Луне построить какое-нибудь красное здание. Допустим, источник света включен постоянно и направлен в сторону Луны. В какой-то день месяца у моей суженой начинаются месячные. В этот момент я бегу к источнику света и направляю зайчика/тень на красное здание, в котором сидит другой ученый. И он знает о значении сигнала. По дополнительной освещенности или затемнению он тут же понимает про месячные и получил он эту информацию быстрее скорости света. Каково?
Аноним 18/02/18 Вск 15:44:17  431644
Стикер (0Кб, 512x415)
>>431642
Аноним 18/02/18 Вск 15:46:47  431645
>>431642
>и получил он эту информацию быстрее скорости света
С чего бы это вдруг? С арифметикой не лады?
Аноним 18/02/18 Вск 15:51:45  431646
>>431645
Движение солнечного зайчика быстрее скорости света.
Аноним 18/02/18 Вск 15:53:28  431647
>>431646
>Движение солнечного зайчика быстрее скорости света.
Я понимаю что ты имеешь ввиду. Но тут не эта скорость важна.
Посчитай ручками, если не очевидно где трабла.
Аноним 18/02/18 Вск 15:55:07  431648
>>431647
Ты б лучше рассказал.
Аноним 18/02/18 Вск 16:08:30  431650
>>431646
Таки у тебя с головой не лады, а не с арифметикой. Смотри, выключть источник света еще быстрее чем зайчики двигать. Значит, светишь на луну, как только начинаются месячные - выключаешь свет. Ученый получает сообщение мгновенно. Профит.
Аноним 18/02/18 Вск 18:42:26  431662
>>431608
А из ртути вылетающие из урана куски выбивать водород не будут? А если нагреть?
Аноним 19/02/18 Пнд 00:14:11  431683
>>431650
Хуевый тред какой-то. Не тупых вопросов, а тупых ответов.
Аноним 19/02/18 Пнд 00:26:49  431684
.png (560Кб, 925x475)
Поясните, что значат эти цифры. Я понимаю, что передаточные отношения, но между чем и чем они могут быть? Между рпм на педалях и задней втулкой? Или как?
Аноним 19/02/18 Пнд 00:33:04  431686
>>431684
Все, уже не нужно.
Аноним 19/02/18 Пнд 06:46:27  431692
>>431662
Нет. Могут выбить нейтроны, но это не точно.
Аноним 19/02/18 Пнд 06:51:31  431693
>>431650
Нихуя не мгновенно. Предположим ты в десяти световых годах от ученого, и ты на него светишь. Ты включаешь свет. Но ученый этого не знает, ему кажется, что источник света включен, т.к. на него светит свет десятилетней давности. А о выключенном свете он узнает только через десять лет. С зайчиком та же фигня - сам зайчик добежит почти мгновенно, вот только побежит он с задержкой в десять лет.
Аноним 19/02/18 Пнд 07:56:33  431697
Вопрос про судебную систему, на какой доске лучше спросить ? Помню на первом курсе у меня было правоведение. Нам сказали что суд, при вынесении решения, может опираться на 1) законы, 2) судебные прецеденты 3) я забыл, вот что третье ? Может ли решения суда противоречить закону ?
Аноним 19/02/18 Пнд 14:02:17  431725
>>431693
Хули ты мне это рассказываешь? Объясни гению который вопрос задал изначальный.
Аноним 19/02/18 Пнд 16:13:21  431741
>>431697
присяжные?
Аноним 19/02/18 Пнд 16:15:20  431742
>>431642
>передавать инфу быстрее скорости света
>с помощью света
>подводные?
Аноним 19/02/18 Пнд 23:50:06  431782
>>431693
> вот только побежит он с задержкой в десять лет.
То есть отправивший зайчика увидит его движение как положено, а получатель увидит движение через 10 лет?
>>431742
>тупых вопросов тред
>отвечать вопросом на вопрос
Аноним 20/02/18 Втр 08:53:10  431801
Слууушайте...
Я тут что подумал.
Вот был большой взрыв, всё раскидало, мы видим размеры этого всего и называем это вселенной.
А что если на самом деле "вселенная" в миллиарды раз больше и там постоянно происходят такие большие взрывы и образуют скопления галлактик (то, что мы называем вселенной), но мы их просто не видим из-за расстояния.
Существует такая теория?
Аноним 20/02/18 Втр 09:59:13  431813
>>431642
Уже раз 20 натыкаюсь на вопросы про зайчиков солнечных и не понимаю, почему долбоебы не соображают, что свет перестанет идти(а значит пойдет тень, если уж совсем тупым языком выражаться) не мгновенно а за время S/c
Аноним 20/02/18 Втр 10:04:20  431815
нашёл полкило кварк-глюонной плазмы, каких ништяков можно намутить?
Аноним 20/02/18 Втр 10:53:36  431821
>>431801
ты про сферу хаббла ничего похоже не слышал?
Аноним 20/02/18 Втр 11:16:41  431822
>>431782
Да, представь, что твой свет - поток воды из брандспойта.
Аноним 20/02/18 Втр 11:20:08  431823
>>431801
Взрыв здесь не как взрыв гранаты, а как раздувание каждой точки пространства, собственно, оно продолжается до сих пор, причем, с ускорением (гугли темную энергию). Инфляционная модель БВ предполагает, что такое раздувание могло быть намного быстрее скорости света, так что Вселенная таки может быть в миллиарды раз больше наблюдаемой части.
Аноним 20/02/18 Втр 19:25:59  431889
>>431823
> Инфляционная модель БВ предполагает, что такое раздувание могло быть намного быстрее скорости света
Погоди-погоди. Про "быстрее скорости света" стало известно недавно, потому как далекие на краю жопы галактики так странно ускоряются, что это не сами галактики движутся, а ширится само пространство.
Аноним 20/02/18 Втр 22:14:58  431910
img.png (469Кб, 785x653)
>>429247 (OP)
Если бы разрешение человеческого глаза было 0,5нм без уменьшения поля зрения
что бы с нами стало?
сошли бы с ума?
его можно добиться?
Аноним 20/02/18 Втр 23:07:26  431917
Есть Петя
Петя смотрит на Васька, который летит в космическом корабле
Космический корабль охуенный(на ведре работает), и двигается со скоростью 0.9c
Васек взял фонарик и ебнул фотонами вперёд корабля, получается Петя увидит, что фотоны относительно корабля движутся довольно медленно(лень считать), а вот относительно точки наблюдения Пети они движутся со скоростью с?
Пытаюсь в сто, пока что не выходит, но мне бы подтвердить теорию мою
Аноним 20/02/18 Втр 23:18:27  431921
>>431917
>>431917
ага
Аноним 21/02/18 Срд 00:20:06  431923
>>429247 (OP)
Почему мунин такой умный? Как стать таким?
Аноним 21/02/18 Срд 00:21:05  431924
>>431917
>что фотоны относительно корабля движутся довольно медленно
Нет. Фотоны всегда двигаются со скоростью c
>Пытаюсь в сто
Ты опять входишь на связь? Читай еб твою мать >>431602
Аноним 21/02/18 Срд 03:52:27  431930
Только что в темноте расколол кусок сахара ножом. Восхищен прям
Аноним 21/02/18 Срд 14:01:05  431955
Напомните, почему людишки не вступают в контакт с разумными, но менее развитыми расами, вроде дельфинов и осьминогов?
При этом почему-то ждут, что на их сигналы в космос кто-то должен откликнуться.
Аноним 21/02/18 Срд 14:57:50  431958
>>431955
Умный дохуя дак вступи.
Аноним 21/02/18 Срд 16:23:28  431965
>>431955
И как с ними можно вступить в контакт?
Аноним 21/02/18 Срд 17:41:33  431971
>>431965
Генитально.
Аноним 21/02/18 Срд 17:45:44  431972
>>431917
Движущийся с постоянной скоростью корабль, если убрать ориентиры, неотличим, для пилота, от неподвижного. Фотоны уебут от корабля со скоростью света.
Аноним 21/02/18 Срд 17:53:33  431973
>>431971
Ууууу...
Аноним 21/02/18 Срд 18:00:15  431974
>>431955
>менее развитыми расами, вроде дельфинов
Ну давай разберём по частям всё тобой написанное.
>работать не надо
>еды от пуза
>друзья, компания, социализация, солнышко
>злые обезьяны сидят в Мордоре и считают себя развитыми, въёбывая по 10 часов на ненавистной работе для покупки того же, что мы жрём просто так
Оправдания, манёвры? Развитый он ска.
Аноним 21/02/18 Срд 18:28:04  431975
>>431974
Если захотим отправим всех твоих дельфинов в рыбий рай за несколько дней. Хорошая попытка рыболюб, но нет.
Аноним 21/02/18 Срд 19:08:09  431978
>>431975
Человечество работает малёхо не так.
Никаких "мы захотим" нет. Если ты спизданёшь "а давайте убьём дельфинов", кто-нибудь предложит "а давайте лучше убьём этого клоуна пиздлявого". И тут ты закончишься.
Аноним 21/02/18 Срд 19:36:59  431981
>>431978
Если я стану царьком и захочу то дельфинам пизда.
Аноним 21/02/18 Срд 19:54:14  431983
Можно ли ввести в аминовый спирт алкил, оставив нетронутой гидроксильную группу? Т.е. чисто за счёт отнятия у азота двух атомов водорода? Сумбурно выразился, но как-то так.
Аноним 21/02/18 Срд 20:37:58  431988
>>431981
Вовсе нет, тебя угробят другие царьки.
Аноним 21/02/18 Срд 21:10:54  431992
>>431983
>аминовый спирт
Не существует в природе
Аноним 21/02/18 Срд 21:24:12  431994
>>431992
Аминоэтанол(Который,собственно,меня и интересует) не существует? И прочие аминоспирты?
Тралливалишь, штоле?
Аноним 21/02/18 Срд 21:38:11  431995
image.png (0Кб, 75x52)
>>431983
Как-то так https://innovative.polytechnic.am/files/publication_article/1/15101348851868.pdf
Аноним 21/02/18 Срд 23:16:18  432001
>>429247 (OP)
Почему ученные говорят об ускорении расширении вселенной? Чем дальше галактика, тем сильнее красное смещение. А ничего что мы видим далекое прошлое? Ясень хер, мы будим видеть большое ускорение галактик, чем они дальше, т.к. мы подбираемся все ближе к Большому Взрыву который хорошо так дал пинка галактикам.
Аноним 21/02/18 Срд 23:31:11  432004
17833-img22[1].jpg (33Кб, 720x540)
blob (60Кб, 1200x916)
3540-16[1].jpg (8Кб, 435x265)
slide9[1].jpg (66Кб, 960x720)
>>431983
>>431995
>>431994
Вообще-то подобные реакции называются реакциями дезаминирования.
Не пойму почему для тебя важна -OH-группа, по всей видимости там может быть то угодно у тебя...
Поэтому нашёл какую-то статью тут: http://www.chem21.info/info/487468/
И хотя это и не то совсем, но всё-же пробегись в гугле по этим ключевым словам: дезаминирование аминоспиртов пинаколиновое.
А ещё посмотри на ион тетраметиламмония у холина пик1, и подумай как его отщепить пик2...
Ну и... На пик3, азотной кислотой отщепляется аминогруппа, однако при этом и -OH-группа разрушается.
Ты там хочешь получить H2N-CH2-CH2-CH2-OH ? Это 3-аминопропанол. http://www.himreakt.ru/3860/
А ещё, глянь на бутиламин, если его можно получить из аминоэтанола, то -OH-группа разрушится,
но вместо крайнего метил-радикала -CH3, наверняка можно было бы её потом и вставить.
Аноним 21/02/18 Срд 23:58:20  432006
>>432004
>H2N-CH2-CH2-CH2-OH ?
Нет. Это только начало. Я думал как на место водорода в аминоэтаноле пропихнуть, теоретически,естественно, CH(CH3)2
Чёрт его знает как это сделать, я вот где ни копаю,всё время только один атом замещается.
Аноним 22/02/18 Чтв 00:12:35  432008
17833-img21[1].jpg (24Кб, 720x540)
>>432006
Что ты хочешь получить на выходе?
Я знаю, есть реакция дезаминирования - это микробиологическое дезаминирование, оно ферментативно,
но она эта как как правило проводится с аминокислотами.
Ферменты, катализирующие дезаминирование, называют деаминазами.
В организме человека дезаминирование в основном происходит в печени,
однако глютамат дезаминируется также и в почках.
Аминогруппа, которая удаляется от аминокислот в ходе дезаминирования, превращается далее в аммиак.


А у тебя - аминоспирт.
И учти следующее - при дезаминировании отщепляется молекула аммиака (третья реакция на пикрелейтед), а его химическая формула NH3,
у аминогруппы формула -NH2, и ещё один атом водорода, обычно отщепляется от -OH или -ООН

Аноним 22/02/18 Чтв 00:23:59  432010
>>431988
Мечтай, рыбоголовый.
Аноним 22/02/18 Чтв 11:03:32  432030
>>432008
Диизопропиламиноэтанол.
>аминогруппы формула -NH2, и ещё один атом водорода, обычно отщепляется от -OH или -ООН
Ну вот так всегда.
Аноним 22/02/18 Чтв 11:12:14  432031
>>432008
Я, вообще, хотел сначала воспользоваться тем,что аминоспирты реагируют и как амины тоже. Т.е.,просто хуярить их нужными галогенидами до потери пульса и даже чуть-чуть потом. Но что-то мне подсказывает,что эта хуйня работать не будет.
Аноним 22/02/18 Чтв 11:24:22  432034
Анон, ты тут самый умный, объясни, пожалуйста, что за хуйня со всеми этими обществами плоской земли и прочей ересью.

Это какой-то очень толстый троллинг или какая-то новая мода на целенаправленную дезу в расчете на неграмотную аудиторию? На ютубе огромное количество роликов, причем не все они тупо текст поверх некой картинки, т.е. времени на создание ролика было потрачено намного больше, чем нужно было бы потратить автору, чтобы разобраться в теме и понять, если он действительно такой малограмотный, что он заблуждается.

Аноним 22/02/18 Чтв 12:03:57  432037
>>432031
Амин посерьезнее нуклеофил. Поэтому всё норм. Главное добавь основание чтобы HCl убирать.
Аноним 22/02/18 Чтв 12:09:38  432038
>>432037
Ну заебись тогда.
>Главное добавь основание
Эм... Вот про это я забыл. Бобра тебе. Большого, толстого,пушистого. Чтоб и погладить можно было,и чтоб на зиму не менее кубометра дров нагрызть в лесу мог.
Аноним 22/02/18 Чтв 12:24:10  432039
>>432038
Но вообще я правда не уверен что всё так просто. Поэтому тебе ссылочек принес. Специально посмотрел.
https://pubs.acs.org/doi/abs/10.1021/op025520o
http://www.tandfonline.com/doi/abs/10.1080/00397910902796086
http://www.nrcresearchpress.com/doi/10.1139/v85-048#.Wo6LKoPFLIU
Аноним 22/02/18 Чтв 12:45:12  432040
анон, почему водород в уравнениях записывается как H2, а не просто H?

И почему корзина не может просто так взять и вставить серу впереди водорода(S + H2 = SH2) в реакции H2 + S = H2S ?
Аноним 22/02/18 Чтв 12:48:02  432041
>>432040
Молекула H2
Можешь как хочешь вставлять. Но просто правила есть, по которым выбирают порядок.
Аноним 22/02/18 Чтв 12:49:54  432042
>>432041
>молекула h2
это сорта энергетических уровней, которые не могут быть незаполненными?
Аноним 22/02/18 Чтв 13:55:00  432046
>>430140
Не секрет. Мыр появился из очка Протокоровы.
Защитник !HOHLOVjqt6 22/02/18 Чтв 14:46:47  432049
А я из психушки выписался
Мимо ОП прошлых тредов/I]
Аноним 22/02/18 Чтв 14:51:56  432051
>>432049
Ты поехавший или ради белого билета чалился?
Защитник !HOHLOVjqt6 22/02/18 Чтв 14:55:40  432052
>>432051
Я мать свою порезал
Аноним 22/02/18 Чтв 15:00:45  432054
>>432052
Зачем ты это сделал? Неужели без причинения физического вреда никак нельзя было обойтись?
Защитник !HOHLOVjqt6 22/02/18 Чтв 15:02:11  432055
>>432054
Почему я это сделал? У меня было отчаяние и чувство безвыходности из-за того, что я не мог ни расслабиться ни отдохнуть. А не мог ни расслабиться ни отдохнуть из-за паранойи "Все на меня смотрят". Ко мне в комнату входили по 25 раз в день, я целыми сутками без перерыва был в напряжении. Из-за этого у меня ехала крыша. Из-за того, что у меня ехала крыша, я потерял всю человечность, начал плохо ощущать себя в этом мире (как-будто какое-то кино), поэтому полностью неосознавал что делал, и наконец, я собрал в себе все обиды и все воспоминания по типу - я единожды закрываюсь в комнате, чтобы побыть в тишине и поспать, а батя начинает ломиться в комнату. Я ему открываю и спрашиваю его хули ему тут надо, а он начинает говорить, что мало ли что ему тут надо, да и вообше это его квартира и моего тут ничего нет. Поэтому я не думал, что мою проблему можно решить словами.
Аноним 22/02/18 Чтв 15:05:12  432056
>>432055
Ну ты же понимаешь, что теперь ты будешь изгоем в научном сообществе?
Защитник !HOHLOVjqt6 22/02/18 Чтв 15:14:09  432057
>>432056
Да(
Аноним 22/02/18 Чтв 16:04:39  432061
Где почитать про зависимость гравитации и времени?
Аноним 22/02/18 Чтв 17:02:36  432063
>>432061
Тебе какой уровень?
Аноним 22/02/18 Чтв 17:03:22  432064
>>432063
Сначала для чайников.
Аноним 22/02/18 Чтв 17:03:22  432065
>>432042
Почитай учебник. Да, связанно с уровнями.
Аноним 22/02/18 Чтв 21:42:04  432085
image.png (275Кб, 700x709)
Аноним 22/02/18 Чтв 23:00:49  432090
rwh[1].jpg (89Кб, 480x296)
Gravitatsionnoe[...].jpg (128Кб, 813x610)
1DLExObInXQ199p[...].jpeg (195Кб, 800x768)
giphy[1].gif (1233Кб, 500x500)
>>429247 (OP)
Почему на картинках с изображением гравитационного искривления пространства-времени,
расстояние между кругами по мере удаления от массивного объекта, либо остаётся таким же (пик1, пик2),
либо уменьшается даже (пик3)?

Прочитал статью про квазары: https://ru.wikipedia.org/wiki/Квазар
и вижу, что аккреционный диск квазара имеет космологическое гравитационное красное смещение.
Здесь, https://ru.wikipedia.org/wiki/Гравитационное_красное_смещение
я вижу, что гравитационное красное смещение проявляется по мере удаления от массивных объектов,
а свет, приходящий из областей с более слабым гравитационным полем (в более сильное) -
испытывает гравитационное синее смещение.
Т. е. гравитация втягивает концентричные круги в себя, и более правильно искривляется пространство-время на 4-й гифке.
Т. е. по мере удаления от центра силы тяжести, километр становится всё длинее и длинее.
Аноним 23/02/18 Птн 07:14:48  432109
Почему так нельзя?
-4^2=(-1)(4^2)=(-1)(4*4)=-16
Аноним 23/02/18 Птн 07:19:24  432110
>>432109
Все понел -1*(-1) чет затупил
Аноним 23/02/18 Птн 12:26:26  432122
Поясните на пальцах про виртуальные фотоны, нулевые колебания и прочую фигню вне массовой поверхности.
Аноним 23/02/18 Птн 12:45:18  432123
Как научиться ориентироваться в химии? Ну ок, несложно разобраться в строении атома и взаимодействии электронов. Но как ориентироваться в туевой хуче соединений и их свойствах?
Аноним 23/02/18 Птн 14:23:28  432129
>>432123
Читай теорхим. Тебе нужен переход от
орбитали--->молекулярные орбитали----->свойства

Советую учебник Хьюи
Аноним 23/02/18 Птн 19:58:34  432141
Поясните за время.
>Это произошло около 2000 лет назад
Что это значит? Т.е. я правильно понимаю, что это произошло около 2000х365 заходов назад?
Аноним 23/02/18 Птн 20:08:01  432143
Как "а" вынести за скобки:
a/b+b/a=a(... )
Возможно ли это?
Аноним 23/02/18 Птн 20:52:26  432145
>>432143
a/b + b/a = (aa + bb)/ba = a(aa + bb)/baa = a((aa + bb)/baa) - внутри скобок квадрат а
Аноним 23/02/18 Птн 20:54:54  432146
>>432145
> внутри скобок квадрат а
А надо вынести.
Поэтому мне сдаётся, что это невозможно.
Аноним 23/02/18 Птн 21:52:01  432154
CDataUsersDefAp[...].jpg (104Кб, 1001x1001)
>>432146
a/b + b/a = a(1/b) + ba/aa = a (1/b + b/a)
Аноним 23/02/18 Птн 22:13:47  432157
>>432154
Нет ь
a*1/b + ba/aa = (a/a)/(b/a) + a (b/a)^2/(b/a) = (a/a +a(b/a)^2)/(b/a) = (a(1/1) + a(b/a)^2)/(b/a) = a(1+ (b/a)^2)/(b/a) = ...
Аноним 23/02/18 Птн 22:19:06  432158
>>432154
a×(1/b + b/a) = a×(1/b + b(1/a)) = ab×(1/b^2 + 1/a) = a×(1/b^2 + 1/a)×b = a×(a/b^2 + 1)×b = a×(a×(1/b^2) + 1)×b
если всё правильно, конечно. Но всё-равно внутри скобок - а.
Аноним 23/02/18 Птн 22:23:21  432159
>>432158
Без этого тут точно никак.
Аноним 24/02/18 Суб 01:48:31  432167
>>432159
У тебя там квадратное уравнение получается.
a/b + b/a = a^2/ba + b^2/ba = a^2+b^2/ba
a^2+b^2/ba = x;
a^2+b^2 = xba;
a^2+ b^2 - xba = 0;
a^2 - (xb)a + b^2 = 0;
Вид обычного квадратного уравнения:
ax^2 + bx + c = 0; и x - это "a" у тебя, и оно в квадрате. И ты его вынести за скобки хочешь...
Аноним 24/02/18 Суб 11:08:21  432191
>>432143
Это невозможно, так как это формула квадратного уравнения. И зачем тебе это надо?
Аноним 24/02/18 Суб 16:36:20  432237
>>432191
Мех.характеристика асинхронного движка. M(w)=2Mk/(s/sk+sk/s). В середине s переменная w. S=(w0-w)/w0.
Но это M(w). А я хотел выразить w(M), а для этого надо вынести S.
Аноним 24/02/18 Суб 20:38:31  432252
Почему законы Вселенной именно таковы какими они являются? Откуда они взялись? Они вроде не однородны, в микромире одни законы, в макромире другие, в мегамире тоже другие? Меняются ли они? Есть ли наука изучающая законы вообще?
Аноним 24/02/18 Суб 23:39:13  432262
>>432252
> Есть ли наука изучающая законы вообще?
Теоритическая физика.
Отвечает на вопросы "а что будет, если законы такие?"
Современное состояние такое. Законы макромира выходят из законов микромира.
Так же и из законов мегамира.
Но мегамира законы плохо стыкуются с закономи микромира, проблема в математике.
Либо аппарата нет. либо данных.
Нужные данные можно получить из областей с черными дырами.
Законы не меняются. Вроде как. Но это не точно.
Вообще наука отличается тем, что к любому утверждению между строк написано "если постулаты верны" А на самих постулатах, "но это не точно".
Например скорость света или симметрия пространства постулируются.
Аноним 24/02/18 Суб 23:44:43  432263
Какова минимальная температура плазмы? Теоретически и практически.
Ну тоесть вот есть твердь, жидкость и газ, при определенных условиях все это можно охладить до околонуля или же разогреть до 9000 градусов. При атмосферном давлении так так же существуют вещества, находящиеся при околонуле в этих трех состояниях. А что же с плазмой? Есть ли вещества, находящиеся в этом состоянии при температуре <= комнатной или даже околонуля?
Аноним 25/02/18 Вск 00:15:20  432265
>>432262
Законы макромира произошли от законов микромира а законы мегамира - от макромира? Они же вроде отличаются, как так? Откуда взялись законы микромира?
>Отвечает на вопросы "а что будет, если законы такие?"
Что это значит?
Аноним 25/02/18 Вск 00:33:30  432266
>>432252
Эт уже философия. Я как-то тоже спросил "что такое информация?", ответ оказался не таким очевидным.
Аноним 25/02/18 Вск 00:58:43  432268
15002-7[1].jpg (22Кб, 441x451)
image6[1].gif (29Кб, 550x190)
>>432263
Самая низкотемпературная и низкоионизированная (порядка 1%) плазма характеризуется температурой до 100 тысяч градусов.
В земных условиях такую плазму можно наблюдать в виде молний.
Температура канала молнии может превышать 30 тысяч градусов, что в 6 раз больше, чем температура поверхности Солнца.

http://femto.com.ua/articles/part_2/2505.html
Низкотемпературной называют плазму, у которой средняя энергия электронов меньше
характерного потенциала ионизации атома (< 10 эВ); темп-pa её обычно не превышает 105К.

Характерным примером неравновесной плазмы является плазма тлеющего разряда или плазма дугового разряда низкого давления;
например, в плазме гелий-неонового лазера при давлении газа ~10 тор (мм рт. ст.)
температура газа в центре разрядной трубки тогда как средняя энергия электронов несколько эВ (пикрелейтед).

Чтоб получить плазму — надо ионизировать атомы. У каждого химического элемента есть потенциал ионизации. Пик2.
В случаях выше - плазма есть, но она низкоионизированная, один или несколько электронов отлетают от атома, получается ион, и ионизация.
Иногда в научной литературе пламя относят к «холодной/низкотемпературной плазме», поскольку фактически оно представляет собой газ,
состоящий из термически ионизированных частиц с небольшой величиной заряда (как правило, не более +/-2-3),
тогда как «истинной», или высокотемпературной, плазмой называют состояние вещества,
при котором ядра атомов и их электронные оболочки существуют раздельно.

Алсо, как вариант можно рассмотреть в качестве низкомпературной плазмы - пары кипящей ионной жидкости.
https://ru.wikipedia.org/wiki/Ионная_жидкость
В настоящее время под термином «ионные жидкости» чаще всего подразумевают соли,
температура плавления которых ниже температуры кипения воды, то есть ниже 100 градусов Цельсия.
Например, у эвтектической смеси LiCl/AlCl3, т.пл +144 градуса. Это ионная жидкость - т. е. раствор состоящий из ионов.
Пары её должны бы давать плазму, если конено ионная связь не настолькоп прочна, чтоб объединить ионы в молекулы.
Аноним 25/02/18 Вск 01:39:45  432270
>>429247 (OP)
Планирую увеличить скорость света в три раза, какие камни?
Аноним 25/02/18 Вск 02:13:38  432274
>>432270
Световой конус.
Аноним 25/02/18 Вск 02:17:22  432275
>>432274
Как раз
Аноним 25/02/18 Вск 02:21:03  432276
>>432270>>432274
>Световой конус
https://geektimes.ru/post/241008/
Вылетая из конденсата Бозе-Эйшнтейна на видео здесь: https://youtu.be/EK6HxdUQm5s?t=54s
свет увеличивает скорость распространения в сплошной среде с 20 км/ч до околосветовой.
При этом его скорость увеличивается во столько раз: https://www.google.com/search?q=скорость+света+%2F+20км+%2F+ч
Т. е. его скорость сначала замедлилась в 53 962 642.4 млн раз, а потом во столько же раз увеличилась, примерно.
Всё - в пределах светового конуса.
Аноним 25/02/18 Вск 02:51:55  432278
>>432268
>ионная жидкость
Как правило при переходе в газ, там либо образуются ковалентные молекулы, либо отдельные нейтральные атомы
>>429247 (OP)
Почему не существует безотходного организма? Например нашем отходом жизнедеятельности является мочевина, но есть бактерии перерабатывающие мочевину в нитраты, а нитраты в ништяки перерабатывают растения. Что мешает добавить уже эти технологические/химические цепочки в наш организм, чтобы был замкнутый цикл азота? Почему эволюция за 4 лярд лет такого не сделала?
Аноним 25/02/18 Вск 03:03:20  432279
>>432237
Нихуя не понятно. Ты хочешь выразить w или w0? Здесь нашёл другие формулы:
http://electricalschool.info/elprivod/1449-mekhanicheskaja-kharakteristika.html
Попробуй выразить s как корень квадратного уравнения...

M(w)=2Mk/(s/sk+sk/s); | × (s/sk+sk/s)
M(w)(s/sk+sk/s)=2Mk; | / M(w)
s/sk+sk/s = 2Mk/M(w); | дроби слева - к общему знаменателю
s×s/s×sk + sk×sk/s×sk = 2Mk/M(w); | запишу в виде квадратов
s^2/ssk + sk^2/ssk = 2Mk/M(w); | сумму квадратов - в числитель
(s^2 + sk^2)/ssk = 2Mk/M(w); | × ssk
(s^2 + sk^2) = 2Mk/M(w)×s×sk; | убираю скобки.
s^2 + sk^2 = 2Mk/M(w)×sk×s; | переношу всё в левую часть
s^2 - 2Mk/M(w)×sk×s + sk^2 = 0; | привожу к квадратному уравнению со стандартными коэффициентами
s^2 + (-2Mk/M(w)×sk)×s + sk^2 = 0;

Итак... ax2 + bx + c = 0 - квадратное уравнение.
Общая формула для нахождения корней квадратного уравнения: x1,2 = -b±√(b^2 - 4ac)/2a;

s^2 + (- 2Mk/M(w)×sk)×s + sk^2 = 0;
Общая формула для нахождения s (подставляю коэффициенты): s1,2 = -(-2Mk/M(w)×sk) ± √((-2Mk/M(w)×sk)^2 - 4×1×sk^2)/2×1;
s1,2 = +2Mk/M(w)×sk ± √(2Mk^2/M(w)^2×sk^2) - 4×sk^2)/2; - вот тебе твоё s.

>В середине s переменная w. S=(w0-w)/w0.
S=(w0-w)/w0;
S×w0=w0-w;
S×w0+w=w0;
w=w0 - S×w0; и там где s - вот это вот усё... С одним, походу, корнем.
Аноним 25/02/18 Вск 16:27:07  432336
>>432278
>Почему эволюция за 4 лярд лет такого не сделала?
Потому что замкнутый цикл лучше работает на всей экосистеме, а не одном организме.
Потом, ты забыл про говно, в котором животные и первобытные люди оставляли семена, из которых вырастали новые растения и деревья - и это тоже часть биологического цикла.
И, конкретно с людьми, ситуация такая, что наш вид уже остановился в эволюции и никогда больше не будет естественных изменений в нашем организме.
Аноним 25/02/18 Вск 21:51:58  432360
Какой организм на земле из млекопитающих рекордсмен по росту? В том плане, что проходит путь от зародыша до взрослой особи быстрее всего?
Аноним 25/02/18 Вск 22:44:08  432362
>>432278
>Почему не существует безотходного организма?
Ну, если что-то входит в организм, то что-то должно оттуда выходить же.
Вот оно это что-то не просто выходит, а как-бы отходит от организма, в качестве отходов.
>Например нашем отходом жизнедеятельности является мочевина,
>но есть бактерии перерабатывающие мочевину в нитраты, а нитраты в ништяки перерабатывают растения.
Прочитал в википедии - там три различных привращения, два из которых производят разные виды бактерий.
1. Бактерии выделяют фермент уреазу, который катализирует превращение мочевины в аммиак и углекислый газ.
2. Аммиак далее окисляется бактериями рода Nitrosomonas в нитрит.
3. Далее бактерии рода Nitrobacter окисляют нитрит в нитрат.
4. Растения поглощают из почвы ионы аммония и нитрат-ионы.
Но как в организме эти бактерии держать, их же не на поверхности тела и не внутри же носить, как бифидубактерии, бля...

Но можно было бы исследовать процессы в этих бактериях, поизвлекать из них ферменты,
и реализовать при помощи тех же механизмов - некий бионический орган, который не выводил бы мочевину из организма,
а разуплотнял бы её последовательными реакциями внутри в различные нитраты и нитриты,
или включал бы аммиак в органику, и синтезировал бы всякие амиды, амины или проводил бы аминирование
всяких там карбоновых кислот извне - с аминокислотами на выходе. Аминокислоты уже более полезны организму.
Но карбамид сам по себе - он же сдерживает и связывает аммиак внутри себя!
Сначала думал через трубочку выводить аммиак - как-бы попёрдывая ним, но затем вспомнил о его высокой растворимости...
И да, ПДК у аммиака же 20 мг/м3 — IV класс опасности!.. Поэтому, свободный аммиак в организме нафиг не нужен,
а нитриты и нитраты бактериями образуются уже после - через промежуточное образование свободного аммиака.

Можно было бы какое-то питательное вещество из него синтезировать и отложить в бионическом органе, как гликоген в печени,
с омыванием органа кровью и высасыванием вещества с его поверхности... Но на синтез чего-то питательного - надо энергия, как и у растений...
И вообще карбамид является конечным продуктом метаболизма, т. е. молекула безполезная, с ней организм ничего не может сделать,
и переварить её не может, и энергию извлечь. Думаю что и бактерии, которые используют фермент уреазу, для ращепления карбамида
тоже "не желали бы" ращеплять карбамид, т. е. они не берут с этого энергии (или всё-же берут?), и этот фермент позволяет им существовать в среде мочевины,
как-бы защищая их, выгоняя аммиак извне в ущерб энергозатратам на его ращепление. Или же просто работает как наследственный механизм для адаптации...

И вообще, бактерии могут передохнуть, а вот орган бионический, напичканый ферментами, мог бы быть ещё и самовосстанавливающимся,
если реализовать в нём некие методы саморегуляции. Например запхнуть ферменты как катализатор на нанотрубки,
и реализовывать регулярное омывание котализатора кровью или плазмой крови,
с последующим наращиванием слоя питательного вещества из карбамида на ферментном катализаторе,
и выдавливанием его за пределы катализатора, через поры в сам орган и его некий мешочек,
то есть уже в ткани его для вымывания кровью - как например происходит наращивание гликогена в тканях печени...

Если можно было бы внутри органа - аминокислоты синтезировать, то наверняка можно было бы из них и белки определённые крутить
внутри органа - как-бы технологически. Например, реализуя внутри процесс трансляции белков
транспортными РНК на отдельных специальных матрицах, как в рибосомах.
Согласись, намного лучше было бы синтезировать жизненно-важные белки программно, нежели в результате эволюции организма.
Аноним 27/02/18 Втр 10:43:31  432469
>>432034
Привлечение внимания.
Аноним 27/02/18 Втр 10:56:35  432470
>>432141
Год = оборот земляшки вокруг солнца, и равен 365,2425 оборотам земли вокруг своей оси (дням).


Аноним 27/02/18 Втр 10:59:56  432471
>>431403
>война конечно дала толчек научному прогрессу
Война это двигатель прогресса
Без нее сейчас никто ничего не производил бы, денег на науку не выделялось бы
Аноним 27/02/18 Втр 11:24:37  432477
1519653808316.jpg (102Кб, 800x600)
>>432471
тракторы таки раньше чем танки сделали
Аноним 27/02/18 Втр 19:25:24  432517
>>429247 (OP)
Исходя из каких соображений в вычислении определенного интеграла методом трапеций/прямоугольников нужно выбирать число разбиений n кроме как "побольше шоб хватило", если эталонного значения (через формулы интегрирования) у меня нет? Можно как то проанализировать рост функции, чтобы сделать вывод типа вот при 11 отрезках мы проебем участок резкого роста, а при 13 будет заебись?
Аноним 27/02/18 Втр 19:41:58  432518
>>432056
>Ну ты же понимаешь, что теперь ты будешь изгоем в научном сообществе?
Лол, схуяли? Или учёные признают только опущенных хуесосов, молча мирящихся с условиями жизни опущенного стадного скота и соседством с омерзительным стадным скотом?
Аноним 27/02/18 Втр 21:44:43  432525
Вопросы по излучению Хокинга и чёрным дырам.
Если суть этого излучения-появление частицы и античастицы по разные стороны границы горизонта событий, то:
1)Почему при этом вообще должно прои
сходить "испарение" чёрных дыр, а не появление в среднем одинакового количества частиц и античастиц за и до горизонта событий?
2)Как понимаю, частица и античастица возникают КРАЙНЕ близко друг к другу, выходит, чтобы возникшей частице улететь, нужно возникать сразу движущейся или со скоростью света, или крайне близко. Т.е. это может быть только свет.
Почему тогда про излучение Хокинга говорят оно состоит(потенциально) преимущественно из фотонов, но не только лишь из них?

И два вопроса о чёрных дырах в целом.
Т.к. на свет не действует гравитация, ввиду безмассовости, из чёрных дыр он не вылетает не потому что гравитация настолько сильная, что вторая космическая скорость становится выше скорости света, а потому что свет движется по круг бесконечно искривлённого пространства. Выходит... тут хотел написать простую херню типа весь свет который когда либо летал в чёрной дыре в итоге долетит до края и будет крайне сконцентрирован сразу же за горизонтом событий, но после того как набрал словосочетание "бесконечно искривлённое пространство"-поплыл и полезла всякая куча мыслей вроде.. ну пиздец, пока записывал это подступление к ним все вылетели из головы.
и второй вопрос-Как чёрные дыры вообще могут что-то втягивать(с точки зрения стороннего наблюдателя) ? При большой гравитации время замедляется, практически останавливаясь возле горизонта событий, выходит всё что чёрная дыра потенциально могла втянуть в себя будет приближаться к ней, замедляясь всё сильнее подлетая всё ближе, но втянуться так и не сможет?
Аноним 27/02/18 Втр 22:36:21  432527
Если сила гравитации уменьшается в квадрате согласно увеличению расстояния, то почему радиус Шварцшильда увеличивается нелинейно-возрастающе при увеличении массы ЧД?
(Понятно что это всё можно увидеть из уравнений, а словами и кратко-понятно распишите плиз кто-нибудь, никак не могу в это въехать, неувязочка какая-то для меня это, сильно давящая)
Аноним 27/02/18 Втр 23:13:39  432530
post[1].gif (25Кб, 600x300)
c38eb1d8ed5c6ef[...].jpg (20Кб, 300x169)
2905[1].jpg (65Кб, 480x296)
7EqX1F20HG8[1].jpg (44Кб, 604x334)
>>432525
>Почему при этом вообще должно происходить "испарение" чёрных дыр,
>а не появление в среднем одинакового количества частиц и античастиц за и до горизонта событий?
Ну так горизонт событий - это граница, которая разделяет пространство с которого что-то может вылететь,
и пространство с которого что-то вылететь уже не может.
Если рождение пар происходит и одна частица падает туда, как она может вылететь? Пикрелейтед.

С другой же стороны, при рождении пар, у каждой из частиц в паре частица и античастица - имеется масса,
и чёрная дыра могла бы поглощать сколь угодно античастиц из рождающихся пар из вакуума, и излучать "частицы" испарением.
Или не обязательно одни лишь античастицы, но и частицы, они внутри чёрной дыры аннигилировали бы а образующиеся фотоны - поглощались бы.
Так не только росла бы масса чёрной дыры - из нихуя, но эта ЧД ещё и была бы источником материи,
в виде разлетающейся извне горизонта событий во все стороны и материи и антиматерии.
Но это всё упрощенно и тупо. Там формулы есть.

https://ru.wikipedia.org/wiki/Излучение_Хокинга
>Также этот процесс очень грубо можно представить как «заём» энергии вакуумом у внешнего поля для рождения пары частица+античастица.
>В отсутствие чёрной дыры аннигиляция «возвращает» энергию полю.
Т. е. как-бы сначала было 0 Джоулей, при рождении N пар - стало минус 1 джоуль, при аннигиляции всех частиц и античастиц у этих пар - суммарно опять 0 джоулей.
>В описываемом случае при наличии чёрной дыры аннигиляции не происходит,
>одна из частиц улетает к наблюдателю, унося часть «занятой» энергии, тем самым уменьшая энергию, и следовательно массу чёрной дыры.
А тут типа - отрицательная энергия поля забирает массу у ЧД.
>2)Как понимаю, частица и античастица возникают КРАЙНЕ близко друг к другу,
>выходит, чтобы возникшей частице улететь, нужно возникать сразу движущейся или со скоростью света, или крайне близко.
>Т.е. это может быть только свет.
>Почему тогда про излучение Хокинга говорят оно состоит(потенциально) преимущественно из фотонов, но не только лишь из них?
Наверняка могут быть ещё глюоны и возможно - гравитоны, они тоже безмассовые, а значит - со скоростью света движутся.
https://ru.wikipedia.org/wiki/Безмассовые_частицы#Известные_безмассовые_частицы
>Т.к. на свет не действует гравитация, ввиду безмассовости, из чёрных дыр он не вылетает не потому что гравитация настолько сильная,
>что вторая космическая скорость становится выше скорости света, а потому что свет движется по круг бесконечно искривлённого пространства.
Оно ещё и закручиваться может это пространство там, как на пик3. Ну и время замедляется, а если оно замедляется, фотон по-сути останавливается.
Он не может существовать без движения, и по всей видимости, его инертная масса (разница масс, обусловленная наличием импульса) - переходит в массу,
после чего - он тупо падает в ЧД, как вот этот чёрный шарик на пик1. Не знаю, возможно ли такое, но звучит прикольно - фотонные микро ЧД.
>Выходит... тут хотел написать простую херню типа весь свет который когда либо летал в чёрной дыре
>в итоге долетит до края и будет крайне сконцентрирован сразу же за горизонтом событий,
>но после того как набрал словосочетание "бесконечно искривлённое пространство"-поплыл и полезла всякая куча мыслей вроде..
>ну пиздец, пока записывал это подступление к ним все вылетели из головы.
Лол. А ты по картинкам в гугле пробегись, если чё понравится, можешь тут запостить. Вот нашёл пару пикч тебе.
>и второй вопрос-Как чёрные дыры вообще могут что-то втягивать(с точки зрения стороннего наблюдателя) ?
>При большой гравитации время замедляется, практически останавливаясь возле горизонта событий,
>выходит всё что чёрная дыра потенциально могла втянуть в себя будет приближаться к ней,
>замедляясь всё сильнее подлетая всё ближе, но втянуться так и не сможет?
Аххах. Вот про этот гон я выше и написал. А только сейчас прочитал. Лол.
Аноним 27/02/18 Втр 23:32:45  432532
KMO090317005151[...].jpg (127Кб, 968x544)
blob (161Кб, 607x597)
14960981299861[[...].jpg (77Кб, 500x492)
>>432525
>и второй вопрос-Как чёрные дыры вообще могут что-то втягивать(с точки зрения стороннего наблюдателя) ?
>При большой гравитации время замедляется, практически останавливаясь возле горизонта событий,
>выходит всё что чёрная дыра потенциально могла втянуть в себя будет приближаться к ней,
>замедляясь всё сильнее подлетая всё ближе, но втянуться так и не сможет?
Если так смотреть, и глубже подумать, то ЧД вообще не в состоянии что-либо затянуть в себя.
Она лишь притягивает материю, свет и пространство - ближе, и всё вокруг неё - становится массивнее от этого.
Она может выглядеть как сингулярность извне, локально вместив в себя всю Вселенную,
но при этом внутри - чёрной дырой не являясь. Лол. Там просто ужатое пространство. И гравитация не действует между объектами внутри.
Представь себе, в 1 планковском объеме - ужат 1 парсек, а за 1 парсек гравитация между объектами падает существенно.

Где-то я уже видел подобное, кажется в теории эонов Пенроуза.
У них там вроде, допускаются незаметные большие взрывы в цикличной Вселенной,
и время циклично ускоряется до бесконечности с постоянным расширением пространства и эволюцией Вселенной в цикле.
https://ru.wikipedia.org/wiki/Конформная_циклическая_космология
Всё это продолжается бесконечно - зацикленными конформными преобразованиями пространства-времени.
Тут уже подключается такой раздел математики, как конформная геометрия. Пик3.
Конформное отображение — отображение, сохраняющее форму бесконечно малых фигур.

А у меня - наоборот, не расширение а ужатие в ЧД, с обнаружением целой Вселенной внутри неё.
Аноним 27/02/18 Втр 23:53:21  432533
>>432530
По первой части-я понимаю принцип, если не вдаваться в то что я спросил, а вопросом я имел ввиду что-то вроде подбрасывания монетки, при стремящемся к бесконечнсти количестве подбрасываний, будет соотношения орёл/решка стремящееся ровно к 50/50
>они внутри чёрной дыры аннигилировали бы а образующиеся фотоны - поглощались бы.
разве при этом образуется фотон? Ничего же не должно остаться при этом.

Про другие частицы понял.

>Вот нашёл пару пикч тебе.
спс
Аноним 28/02/18 Срд 00:04:30  432534
>>432532
>Если так смотреть, и глубже подумать, то ЧД вообще не в состоянии что-либо затянуть в себя.
>Она лишь притягивает материю, свет и пространство - ближе, и всё вокруг неё - становится массивнее от этого.
Так вот весь проблемес в том, что много где(в научпоп видосиках на ютубе, кек) вижу заявления что "чёрная дыра поглотила другую чёрную дыру" "чёрная дыра поглотила-то-то" "столкнулись/слились две чёрные дыры" как о свершившемся факте, т.е. а о том что это невозможно нигде упоминаний не нашёл и близко, будто это сам-собой разумеющееся, типа "лол, ну это же чёрная дыра, она всё всасывает)) она и должна это делать))"
причём не просто как о возможном событии, а как о в принципе довольно быстро происходящем событии
Аноним 28/02/18 Срд 01:01:10  432535
blob (45Кб, 225x224)
radiometermovin[...].gif (199Кб, 200x198)
201307031704031[...].gif (779Кб, 299x360)
>>429247 (OP)
Один анон написал пост здесь https://2ch.hk/sci/res/432515.html
но тред прикрыли, поэтому отвечу ITT с цитатой:
>Если бы в макромире ВООБЩЕ не существовало трения и тепловых потерь - всё бы кругом работало по циклической инерции, всегда рекуперируясь на 100%.
>В микроуровне всё так и работает, там трения нет, и изолированное от теплообмена нагретое тело будет оставаться таковым бесконечно долго.

>Поскольку второе начало термодинамики сформулировано для молекулярной материи, то в халяве на макроуровне всё упирается в демона максвелла,
>состоящего из чего-то меньше молекул. Такой демон максвелла должен использовать на один такт значительно меньшую энергию
>по сравнению с сепарируемой энергией молекулы.
>Из чего и как такую машину строить - пока не вполне понятно, пока что известные нам крупномасштабные немолекулярные структуры
>есть только в нейтронных звёздах.

>Теоретически можно наебать и первое начало термодинамики, поскольку на субмикроуровне есть флуктуации пространства,
>которые мы пока умеем только чуть-чуть подавлять вакуумом Казимира.
>Если каким-то хуем сепарировать их - можно было бы получать бесконечное количество энергии из нихуя
>(ну может не совсем из нихуя, а из неведомых совсем запредельных процессов в многомерных бранах),
>и противоречия с научной картиной мира в этом бы не было.
>Но из чего делать сепаратор для субмикроуровня - это вообще тайна великая есть,
>требуется манипулирование веществом на околопланковских масштабах.

>>432529
>Такой демон максвелла должен использовать на один такт значительно меньшую энергию по сравнению с сепарируемой энергией молекулы.
>Из чего и как такую машину строить - пока не вполне понятно, пока что известные нам крупномасштабные немолекулярные структуры есть только в нейтронных звёздах.
Не для квантового уровня конечно, но в макромасштабах может и сошло бы.
Теоретически в качестве демона Максвелла - можно было бы использовать Демона Фейнмана (храповик и собачка), но в плазме, с разноимённо заряженными лопастями.
Что-то типа электрического радиометра Крукса в плазме, только электростатического.
Хаотичное движение - упорядочивается электрическим полем, столкновения ионов с одной стороны упругие, а с другой - нет, и собачка на храповике.

Напряжённость электрического поля на разноимённо-заряженных пластинах должна быть настолько большой,
чтобы ионизировать газ и поддерживать плазму, порождая направленый ток ионов, ядра которой обладают кинетической энергией.

В общем, смотри... Пластина имеет две поверхности: белая - положительно заряженная, и чёрная - отрицательно заряженная.
1. белая поверхность пластины с положительным зарядом, в которую влетает молекула (газ), атом (инертный газ), ядро атома (плазма)
или неполностью ионизированный ион (низкоионизированная плазма, например) - эта поверхность своим мощным положительным зарядом,
отнимает все электроны у влетающего атома, а само ядро ядро атома - выталкивает из поля.
Ядро, обладающее массой, упруго отскакивает взаимодействуя с полем этой поверхности у пластины, и возможно даже тем самым, давит на неё.
Но такой механизм как собачка https://www.youtube.com/watch?v=rWPC28xlrtE препятствует передачи импульса от ядру к пластине,
поэтому происходит упругое отскакивание ядра, с сохранением кинетической энергии ядра, возможно даже без взаимодействия и передачи импульса.
Т. е. пластина может не нагреваться от этого.
2. чёрная поверхность пластины с отрицательным зарядом - напротив втягивает в себя ядра и атомы, выдавливая электроны за пределы его.
Образующийся электронный газ - прёт на белые поверхности, порождая электронный ток.
Электроны обладают малой массой, поэтому передают незначительный импульс белым поверхностям.
Вся кинетическая энергия содержится в массивных ядрах ионов, хаотично движущихся за пределами пластин и поверхностей,
и движущихся направленно - в электрическом поле их.
Дальше, чёрная поверхности имеет отрицательный заряд. Она притягивает эти ядра ионов, затягивая их в себя.
Они соударяются с кристаллической решёткой черной поверхности,
и передают импульс и создают давление вращая храповик (собачка не мешает в этом направлении).
А ещё эти энергичные ядра ионов нагревают эту поверхность пластины.
Но ядра при этом - не отскакивают унося часть импульса, а продолжают притягиваться отрицательным полем, продолжая при этом передавать импульс.
И так - пока не передадут весь импульс. Материал чёрной поверхности пластины облепляется ионами и ядрами,
они перестают двигаться хаотично, все колебания гасятся наличием сильного электрического поля. Они липнут упорядоченно.
Таким образом, на поверхности чёрной пластины появляется структура из совокупности ядер, или слой из положительных ионов,
сама поверхность охлаждается (ну не двигаются атомы кристаллической решетки и сами ядра хаотично - они там тупо прилипли),
и образуется некий положительный конденсат из атомов.
А потом, внезапно, падает напряжённость отрицательного поля, например из-за срабатывания собачки, когда на пластину уже дохрена налипло ядер.
Атомы, облепившие чёрную поверхность, при снижении напряжённости поля, берут электроны из этой пластины,
и отщепляясь от поверхности этой чёрной пластины, падают, собираясь в ёмкость
или выдуваются потоком горячих ионов, со снижением температуры горячих ионов и раскачиваются сами теплообменом от них.

В общем, как-то так при постоянной температуре газа или плазмы можно получить атомы с меньшей кинетической энергией,
а значит температурой, упорядочивая хаотичное движение и разделяя потом на разные вещества - холодные и горячие атомы.
Менее холодный газ или плазма из охлаждённых атомов - может идти в нагреватель или в плазмотрон, порождая снова изначальную энергичную плазму.
Первая белая поверхность пластины могла бы иметь в своём составе микроплазмотроны (вольфрамовые иглы) >>430618
и состоять из них - т. е. тоже могла бы ускорять атомы, но для этого надо переменое положительное электрическое поле на этой пластине.
Регуляцию его интенсивности можно было бы замкнуть на собачку в храповике. Всё это могло бы ещё и резонировать. Ядерный магнитный резонанс есть же.
Но не думаю, что это имело бы смысл - снова разогревать уже холодный газ и доводить его до состояния плазмы,
ведь демон Максвелла призван разделить холодные и горячие молекулы, и собрать их отдельно для последующего и закономерного теплообмена между ними.
Поэтому на вход могла бы подаваться горячая низкоионизированная плазма, а тепло всё - превращаться в механическое движение пластин и храповика,
с холодным газом на выходе, или вообще сжиженным конденсатом. Что-то типа вечного двигателя второго рода.
Особенно если напряжённость полей менять циркуляцией зарядов без потерь - в сверхпроводниках.
Возможно даже так, можно было бы извлечь и тепловую энергию из атмосферного воздуха, с его температурой +20°C
сжижив его в результате и растворив потом в жидком азоте замёрзшие кристаллы разных газов, лол.
Но атомарный кислород слишком активен, поэтому наверняка можно было бы нормально юзнуть - инертные газы.
Аноним 28/02/18 Срд 01:20:17  432536
>>432535
Смотрю на это так. Какой-нибудь жидкий ксенон (с массивным ядром) стекающий со второй чёрной пластины,
разогревается окружающей средой, и испаряясь приобретает кинетическую энергию - отнимая тепло от окружающей среды.
Дальше он ионизируется, ядра отскакивают в сосуде хаотично, и вблизи пластин движутся направленно.
Криптон этот или аргон снова конденсируется полем на чёрной пластине и цикл повторяется.
Храповик - крутится вхолостую, собачка щелкает. Всё тепло берётся только с окружающей среды.
Можно подогреть чуток всё это дело, чтоб увеличить интенсивность вращения с нагрузкой храповика.
А можно жидкий инертный газ подать в цилиндр, чтоб он расширяясь там от тепла извне - производил работу, или тупо охлаждал что-нибудь.
Аноним 28/02/18 Срд 01:37:10  432537
>>432535
А ещё можно было бы на чёрную поверхность - микроскопические чёрные дыры напхнуть, ну она же чёрная. Лол.
С одной стороны пластины - упругий отскок положительно заряженных ядер от плюсового поля белой поверхности,
без передачи импульса и энергии этой поверхности, а с другой стороны полное поглощение и приём всей кинетической энергии чёрными дырами.
Разница импульсов на поверхностях, разница давлений по разные стороны пластины, вращение радиометром храповика + собачка на храповике.
Если так, то такую же шнягу можно было бы и на квантовом уровне реализовать.
С одной стороны шкварки летят на белую поверхность, и группируются на разноимённо заряженных иглах,
облепляют их после чего отскакивают от поверхности с кинетической энергией - потому что поле переменное.
С другой стороны, они попадают в короткоживущие чёрные дыры и поглощаются там.
На выходе, после испарения чёрных дыр - в основном фотонное излучение, которое может накачать лазер для воссоздания чёрных дыр.
Оно могло бы кстати и греть кванты всякие эти с газом и порождать дыры, как у этих при облучении йодметана: >>432493,
но если его замкнуть и вывести назад, каким-нибудь изогнутым оптоволокном - в ёмкость с сепарируемыми частицами.
Ну и алсо, сами чёрные дыры можно достать из материала райских птиц: https://mir24.tv/news/16286287/v-peryah-raiskih-ptic-nashli-chernye-dyry
для этого надо как минимум попасть в рай и на потеху Боженьке - поймать такую птицу.
Аноним 28/02/18 Срд 03:09:00  432539
>>432534
Зачем ты вообще его слушаешь, это же очевиднейший поехавший хуесос.
Аноним 28/02/18 Срд 03:42:31  432540
>>432539
Ну не таких же уёбков как ты слушать, которые приходят просто посрать в тред. Вам не место на планете.
Аноним 28/02/18 Срд 07:59:11  432548
>>432525
>Т.к. на свет не действует гравитация, ввиду безмассовости,
Гравитация взаимодействует не с массой, а с энергией. Энергия у фотона есть, т.к. у него есть импульс. Поэтому гравитация вполне нрмально воздкействует на фотон.
Аноним 28/02/18 Срд 12:15:31  432558
8ZNdPLSYif4xbRy[...].jpg (129Кб, 1020x729)
Сап, допустим я сделол частицу, с отрицательной массой, и распидорил её до скорости, превышающей скорость света. Какие практические профиты это даст нам, в плане передачи информации? Какие парадоксы возникнут?
Аноним 28/02/18 Срд 12:42:12  432562
>>432558
Парадокс не сделанных уроков.
Аноним 28/02/18 Срд 14:25:42  432578
>>432562
Это уже давно не смешная и избитая фраза. Очевидно, что тебе сказать нечего, но ты жаждешь хоть какого-внимания, ввиду ничтожности и унылости.
Аноним 28/02/18 Срд 14:41:13  432586
https://2ch.hk/sci/res/432585.html
https://2ch.hk/sci/res/432585.html
https://2ch.hk/sci/res/432585.html


Топ тредов
Избранное